Наука

Ответить в тред Ответить в тред
Check this out!
Тред тупых вопросов и острых ответов Аноним 09/10/21 Суб 20:10:03 5471201
image.png 487Кб, 640x382
640x382
Тред тупых вопросов и острых ответов
Аноним 09/10/21 Суб 22:45:28 5471272
А чё, есть прям какой-то резкий переход между длинами волн фотонов, когда вот он инфракрасный свет который поглощают почти все материалы легко и полностью, а вот ещё немного и это уже радиофотоны которые легко проходят через всё кроме металлов?
Аноним 10/10/21 Вск 03:57:10 5471373
>>547086 →
>А тут видите ли акреционный диск ЭТО ДРУГОЕ, лол
Унтерменш, аккреционный диск - это то же самое, когда на Землю падает астероид. Почему он падает? Потому что Земля его притягивает посредством своей гравитации. Падение астероида на Землю - это такая же аккреция, как и падение горячего газа на черную дыру. У тебя Земля тоже засасывает в себя астероид, ебанавт?
>>547081 →
>пук хрюк
>>547087 →
Все ясно, ты ёбаный жирнич. Иди на хуй со своим тухлым траленком.
>>547089 →
>пук
>>547100 →
Потому что при таком сильном искривлении пространства, для того чтобы выбраться из области этого искривления, фотонам пришлось бы двигаться со сверхсветовой скоростью, а это невозможно. Вторая космическая для черной дыры равняется скорости, большей скорости света.
Аноним 10/10/21 Вск 05:08:27 5471394
Я короч охуел что Пи есть в постоянной стефана-больцмана, типа просто излучение, не шара не круга, и каким-то боком Пи, охуеть

Ещё вроде видел в какой-то формуле гравитационную постоянную, в формуле которая ну вообще к ней никакого отношения не имеет, не подскажите в какой?
Аноним 10/10/21 Вск 05:18:24 5471405
Что такое черная дыра? Слышал что черная дыра может быть размером с точку, это как? Понимаю что это сингулярность и не понятно что это такое, но есть ли у науки хоть какие-то предположения и варианты?
Аноним 10/10/21 Вск 06:33:03 5471426
>>547140
>Что такое черная дыра?
Тело, для которого вторая космическая скорость больше, чем скорость света.
>Слышал что черная дыра может быть размером с точку, это как?
Хуй знает, в ЧД никто не заглядывал.
>Понимаю что это сингулярность
Нет.
>но есть ли у науки хоть какие-то предположения и варианты?
Oche плотно сжатый газ.>>547140
Аноним 10/10/21 Вск 07:03:17 5471447
>>547140
>Что такое черная дыра?
Компактный объект, образующийся в результате смерти массивной звезды. Термоядерное топливо заканчивается, давление перестает работать и звезда начинает неограниченно коллапсировать - сжимается в точку с нулевым размером, бесконечной плотностью и бесконечной кривизной пространства (само пространство как бы сворачивается). Эта точка называется гравитационной сингулярностью, а вокруг нее образуется проницаемая в одну сторону граница, которая отделяет сингулярность от внешней Вселенной, - горизонт событий. Все что попадает под горизонт не может оттуда выбраться, включая фотоны, поскольку под горизонтом настолько сильное искривление пространства, что для его преодоления нужно двигаться со скоростью, превышающей скорость света, а это невозможно. Путь попавшего под горизонт объекта лежит только в сингулярность, где он навсегда исчезает из Вселенной.
Аноним 10/10/21 Вск 07:29:34 5471458
Кто хорошо знаком с математикой, расскажите, были ли у вас моменты прозрения в её общем понимании, какие-то важные идеи, без которых как вы считаете нельзя впринципе её понять?
Аноним 10/10/21 Вск 08:45:06 5471479
>>547140
Ну как с точку. Если верить в говне мочоным, наблюдать ее снаружи ты сможешь ровно до сферы шварцшильда, а для солнца это три километра, например. Довольно пухлая точка.
Аноним 10/10/21 Вск 09:18:34 54714810
Едем на велосипеде 20 км/ч
Включаем фонарик
Скорость исходящего света = 20 км/ч + скорость света
Жду оправданий
Аноним 10/10/21 Вск 09:30:14 54714911
>>547148
в среде воздуха скорость смета близко не 300000 км сек а значит эти 20 км в час несущественны
Аноним 10/10/21 Вск 10:13:38 54715012
>>547127
Нет, деление на типы электромагнитного излучения довольно условно. При изменении длины волны свойства фотонов также будут постепенно меняться, резкого перехода не будет.
Аноним 10/10/21 Вск 10:41:10 54715113
image.png 241Кб, 315x465
315x465
>>547148
Прописываю тебе в ебасос и засовываю фонарик тебе в очко. Проблемы?
Аноним 10/10/21 Вск 12:26:46 54715314
>>547150
А похоже что есть довольно чёткая граница
Потому что инфракрасный спектр это довольно широкий диапозон, и его почти все вещества задерживают, а потом вдруг раз, и это уже радио которое через них свободно проходит
Аноним 10/10/21 Вск 12:33:34 54715415
>>547139
Ну там же квантование. Просто если ты что-то решаешь с квантованием, то придется решать волновые уравнения, а там сами собой возникают решения с пи. Посмотри например на частица в ящике, схожая задача, там тоже пи возникает, ибо есть условие, чтобы длины волн укладывались.
Аноним 10/10/21 Вск 12:37:08 54715516
>>547153
произошло диалектическое снятие количества в качество сократ же предупреждал
Аноним 10/10/21 Вск 12:39:50 54715617
>>547154
Этот прав ксати. Пи связывает бесконечно уменьшающуюся меру (центральный угол) с двумя конечными - радиусом и длиной окружности. Хотя окружность тут не при чем. Так устроена Вселенная вашим мамок, малята.
Аноним 10/10/21 Вск 13:25:32 54715818
>>547154
Ну это понятно, но раз даёт ответ похожий на правильный такое решение, то Пи плотно зашито во всю реальность/материю и т.д. а не только в отношение длины окружности к радиусу.


Вот кстати ты про волновые решения говоришь, и раз туда Пи суют, то это например скорее всего значит что фотон это вот реально такая хуёвина как рисуют типа "волна" синусоида-луч.
Аноним 10/10/21 Вск 13:31:10 54715919
>>547158
фотон нарисовать нельзя он безразмерный (в пространственном отношении), его можно только оценить
Аноним 10/10/21 Вск 13:37:25 54716020
>>547159
>он безразмерный
Так чё, были эксперименты типа светили лезером на 1-10км, и быстро открывали/закрывали створки?
3/100000 секунды, бля, быстровато, хз даже есть ли механика работающая на таких скоростях
Ну можно диск с кучей маленьких отверстий быстро крутить
Аноним 10/10/21 Вск 13:41:27 54716121
>>547160
Мало того, что размера нет, так и собственное время фотона - вечность. В этой Вселенной во всяком случае.
Аноним 10/10/21 Вск 13:51:57 54716322
>>547161
Это философия, а эксперимент что?
Вангую опыт с синхронизацией двух таких дисков, а электроника может очень быстрые процессы фиксировать
Вполне точно позволил бы определить например длину фотона.

Раз он такой небацца безразмерный филосовски, но эксперимент показал бы что вот он в метре между двух закрытых створок предположительно было лазерный импульс, а затем детектор показал что он вышел оттуда без ослабления интенсивности, и так происходит 100 случаев из 100, то это вполне ограничение размера фотона.
Т.е. он уже не безразмерный, а "меньше метра".
Аноним 10/10/21 Вск 13:57:56 54716423
>>547163
А ты проведи эксперимент-то, лол. Телками и деньгами завалят, если выйдет по-твоему.
Аноним 10/10/21 Вск 14:01:35 54716524
>>547164
>если выйдет по-твоему
А типо не выходило.

Аноним 10/10/21 Вск 14:03:28 54716625
>>547165
нет в конечно в твоей постановке вообще нет, тебе же нужно будет каждый фотон промаркировать, что бы убедиться, что это ОН
Аноним 10/10/21 Вск 14:05:00 54716726
>>547166
Ты совсем какой-то шизик
Речь о примерном определении размеров, для этого не нужно олин фотон, размер измеряется сразу всего лазерного короткого импульса.
Аноним 10/10/21 Вск 14:07:08 54716827
>>547167
конечно я шизик, лол
эксперимент в студию, фантазер хуев
пиздобол ебаный теоретик двачиный
Аноним 10/10/21 Вск 14:11:41 54716928
>>547168
Хуясе разрыв ФИЛОСОФА, лол
Иди дальше смотри научпоп про фотон который одновременно во всех местах вселенной и при этом для него не существует времени, но при этом бесконечно большое, и при этом она мнгновенно исчезает как только появится "с его точки зрения"
Аноним 10/10/21 Вск 14:13:39 54717029
>>547169
ну так чо, эксперимент будет, лауреат нобелевки без пяти минут хуев
Аноним 10/10/21 Вск 14:30:48 54717230
>>547167
Чел, у тебя какие-то примитивные представления о природе квантовых частиц. Это не шарики, и не волны, а возмущения поля в данном случае э/м.
Аноним 10/10/21 Вск 14:32:21 54717431
>>547170
Их наверняка уже делали.
И всё наверняка так как я сказал. Т.е. с вполне чёткими размерами.
Аноним 10/10/21 Вск 14:33:05 54717532
>>547174
ну иди наверняка получи свою наверняка премию
Аноним 10/10/21 Вск 14:34:08 54717633
>>547175
Так не я ж этот эксперимент провёл, а до меня уже кучу раз куча людей.
Аноним 10/10/21 Вск 14:35:38 54717734
>>547167
Видишь ли в чём дело, квантовая механика накладывает некоторые ограничения. Если ты точно знаешь энергию фотона, то он у тебя имеет бесконечный размер в обе стороны (просто прямая ебучая), если же ты знаешь его размер (ну например условно 20 см), то это уже описывается как суперпозиция состояний с разной энергией. У современных лазеров выходят примерно такого размера волновые пакеты. Вопрос только в том, считаешь ли ты это одним фотоном.
Ну и да, еще кое-что, у тебя у эм поля, существует суперпозиция по количеству фотонов. В итоге лазер тебе выдает такое э-м поле, которое и по энергии немного "размазано" и по количеству фотонов тоже. В итоге одиночный фотон родить не получится. А если порвать жопу и магией родить состояние поля с одним всего фотоном конкретной энергии, то он должен быть бесконечного размера в обе стороны.
Аноним 10/10/21 Вск 14:39:03 54717935
>>547176
ну предъяви эту кучу лол
Аноним 10/10/21 Вск 14:43:25 54718136
>>547179
Два малолетних деграданта, блядь.
Аноним 10/10/21 Вск 14:45:46 54718237
image.png 1299Кб, 895x503
895x503
Аноним 10/10/21 Вск 15:11:47 54718638
>>547179
Так ты не знаешь вообще про них? Т.е. реально зритель научпопа про невероятные квантовые необъяснимые чудеса?
Аноним 10/10/21 Вск 15:16:46 54718839
>>547186
фотон я, без времени и пространства, ибо не могу я зафиксировать протяженность свою и длительность в данных мне в моменте ощущениях
как мал я в пространствe и как долог в настоящем
ибо нет у меня
ничего
только гитлер
Аноним 10/10/21 Вск 15:45:47 54719140
>>547188
+Уважение, анон, давно так не проигрывал.
Аноним 11/10/21 Пнд 01:38:22 54722841
>>547177
Слуш, а вот лазерами металл режут и другие материалами
Вроде самые длинноволновые это СО2, 10600нм(10.6мкм), но эти вроде плохо металл берут, чем твердотельные 1064нм
хз какие самые длинноволновые есть которыми что-то можно резать, но суть в том, что можно ли сделать разрез, тоньше длины волны?

Или опять блять, начнётся, фотон во всех местах во вселенной одновременно... не, не так
ФОТОН ОДНОВРЕМЕННО ВО ВСЕХ МЕСТАХ ВСЕЛЕННОЙ, НО ВОТ ЛАЗЕР ВСЕГДА ЕБОШИТ СВОЙ ТОНКИЙ АККУРАТНЫЙ РЕЗ СО 100% ПОВТОРЯЕМОСТЬЮ, ОХУЕТЬ КАК ЕМУ УДАЁТСЯ ТАК СОВЛАДАТЬ С ЭТИМИ ЕБЛИВЫМИ ФОТОНАМИ КОТОРЫЕ ВО ВСЕХ МЕСТАХ ВСЕЛЕННОЙ УСПЕВАЮТ ПОБЫВАТЬ ЛЕТЯ ОТ ЕГО ВЫПУСКНОГО БЛЯТЬ ОТВЕРСТИЯ И ДО МЕСТА РЕЗА

ФОТОН ОБЕЗУМЕЛ И ШЛЯЕТСЯ ПО ВСЕЙ ВСЕЛЕННОЙ ЗА БЕСКОНЕЧНО МАЛОЕ ВРЕМЯ, И ВООБЩЕ НЕПОНЯТНО В ПРОШЛОМ НАСТОЯЩЕМ ИЛИ БУДУЩЕМ,
СМОЖЕШЬ ЛИ ТЫ СОВЛАДАТЬ С НИМ??
ЛАЗЕР ВОТ МОЖЕТ!
Аноним 11/10/21 Пнд 01:49:38 54722942
Есть какая-то принципиальная сложность концентрировать инфракрасное излучение, тепловое, допустим от предметов +20 градусов, кварцевыми или селенид-цинковыми линзами или просто металлическими отражателями и использовать для повторного нагрева чего-то до температур больше чем предметы которые это излучение инфракрасное излучают?
Я сначала подумал что вроде збс идея и никаких проблем, но потом пришла мысль что эти линзы и отражатели и сами температурой те же +20 градусов, и как-то закрались сомнения не будет ли это мешать...
Аноним 11/10/21 Пнд 02:13:48 54723043
>>547142
>Oche плотно сжатый газ.
Скорее всего там не может существовать никакая материя, т.к. переносчики взаимодействия не могут двигаться быстрее света, и всех их притягивает к центру

Как бы с одной стороны с гравитацией вроде нету такого ограничения, как с другими вещами при приближении к с, типа масса/энергия/импульс будут до почти бесконечности увеличиваться при приближении к с но достичь очевидно не смогут, чтобы притянуть необратимо свет вроде нужна вполне конечная и относительно небольшая СИЛА гравитации, т.е. очевидно чтобы ускорение свободного падения было 300к/сек, не нужна бесконечная сила
а с другой стороны вроде и нужна... хз короче, но тут одна из двух хреней и в любой из них материя не может существовать, или бесконечная сила всё сжимает в нечто непостижимое, или переносчики взаимодействия летят не туда куда надо и это уже не материя

бля, короче очень странный объект, суть в том, что тут опять проблема резкого перехода и квантовой хуйни, типа дискретен мир или нет, вот 299,999,9999999999999999999 это ещё вроде законно и может существовать материя, а потом бац, и ровно 300,000,0000000000, или даже с 1 на конце, и вот когда этот переход? миллионная цифра после запятой? триллионная? типа до этого всё строго отлично, а потом бац, и уже нихуя невозможное состояние
и вот не может же в реальности быть таких резких переходов
Аноним 11/10/21 Пнд 12:06:27 54724544
>>547229
Тепловое излучение в первую очередь равновесное, а это значит как ты не фокусируй его, полезная передача энергия будет возможно только при значительной разности температур фона и предметов.
Аноним 11/10/21 Пнд 12:18:52 54724745
1523239292605.jpg 933Кб, 2208x1242
2208x1242
Помогите с математикой.
Никак не могу понять логику переноса слагаемых(?) в производных функции. К примеру, в y=(10/x^3)-4cosx я получаю y=-(30/x^4)+4sinx, а калькулятор, которым я перепроверяю себя, получает y=4sinx-(30/x^4).
Как это происходит? У меня было несколько наивных предположений, но в ходе работы все они опроверглись.
Аноним 11/10/21 Пнд 12:22:48 54724846
>>547247
Ты дебил? Какой на хрен перенос слагаемых? Ты не знаешь, что сложение коммутативно и a + b = b + a ?
Аноним 11/10/21 Пнд 12:31:17 54724947
>>547247
>математикой
Ты не вт от раздел залетел, петушочек. Брысь нахой!
Аноним 11/10/21 Пнд 12:36:34 54725048
>>547249
Другого не нашёл. В un все на университеты дрочат.

>>547248
Нет, у меня в школе по математике двойка была.

Это я понимаю, но мне неясно почему у калькулятора решение получается в одном виде, а у меня в другом. Зачем он переставляет эти части местами?
Аноним 11/10/21 Пнд 13:03:22 54725149
>>547250
А зачем ты там скобки установил?
Аноним 11/10/21 Пнд 13:49:54 54725450
>>547228
>хз какие самые длинноволновые есть которыми что-то можно резать, но суть в том, что можно ли сделать разрез, тоньше длины волны?
Блять, я когда такое читаю, у меня блять аж кровь уже закипает. Пиздец, что в голове у представляющего фотоны происходит? Типа "длина волны это значит реально волна бежит по пространству, а еще она продольная".
Сука блять, иди нахуй физику учи сначала хотя бы за восьмой класс
Аноним 11/10/21 Пнд 13:52:11 54725551
>>547250
>Зачем он переставляет эти части местами?
Потому что может
Аноним 11/10/21 Пнд 13:55:51 54725652
>>547254
Ну волна это типа как пила, а длина волны - насколько зубья разведены. Видимо.
Аноним 11/10/21 Пнд 15:19:04 54725953
15346556120500.png 180Кб, 492x518
492x518
Аноним 11/10/21 Пнд 15:32:53 54726054
>>547254
Ну вот ты порвался, а по факту тебе ответить нечего.
Аноним 11/10/21 Пнд 17:12:00 54726455
Есть ли тут хоть кто-то, считающий что знает математику действительно хорошо? Кто бы себе честно об этом мог признаться??
Аноним 11/10/21 Пнд 17:41:12 54726756
Я вот что скажу. Астрофизика- лженаука. Никто никогда не видел чёрные дыры. С ними не было поставлено ни одного эксперимента, они не удовлетворяют научный принцип. Но люди продолжают верить другим людям, которые якобы просчитали наличие этих "черных дыр" Более того, эти странные люди даже объединяют их во всякие системы, выдумывают им внешность и названия. Пиздец какой-то, мракобесие. С таким же успехом я могу утверждать что между Землей и Марсом вокруг Солнца по эллиптической орбите вращается фарфоровый чайник и никто не сможет опровергнуть мои слова.
Аноним 11/10/21 Пнд 18:02:30 54726857
>>547267
Антисетипетух, пробуй тоньше.
Аноним 11/10/21 Пнд 22:52:23 54728058
148743844718266[...].gif 3958Кб, 584x640
584x640
>>547267
Двачую, ну вот вращается звезда хуй пойми вокруг чего, что явно в раз 1000 минимум массивнее её, ну не светится оно, ну так светилось бы оно только будь сделано из водорода, а будь сделано из железа или камня то не светилось бы
Где доказательство что это не просто крупное скопление железа и камня?
Аноним 11/10/21 Пнд 22:57:55 54728159
>>547280
А может и светится, прост света такого горнего еще регистрировать не научили моченые.
Аноним 11/10/21 Пнд 23:29:19 54728260
>>547281
А вдруг там всё такое чёрное потому что оно чёрным светится???
Мне кажется логично, ибо какого хуя там всё такое чёрное
Явно в центре мощный источник чёрного излучения
Аноним 11/10/21 Пнд 23:31:03 54728361
>>547282
Все логично, коллега. Светит антифотонами.
Аноним 12/10/21 Втр 04:26:35 54728762
106371512small.jpg 12Кб, 800x450
800x450
>>547267
>Никто никогда не видел чёрные дыры
Ну и нахуя ты пиздишь?
Аноним 12/10/21 Втр 07:15:42 54729463
>>547287
И что это за бублик? Чому такой жолтый внизу?
Аноним 12/10/21 Втр 08:22:29 54729664
>>547294
Это дупло твоей мамаши. Видишь, как сильно оно разъебано ниггерскими хуями?
Аноним 12/10/21 Втр 10:29:42 54730165
Экономит ли энергию расфокусирофка зрения?
Аноним 12/10/21 Втр 11:07:24 54730466
>>547153
Самы оче проникающий инфракрасный не проникает на 0.(0)1 шишечки, а самы оче слабый радиосигнал проникает на 0.(0)1 шишечки, пнятно?
Аноним 12/10/21 Втр 11:15:28 54730667
>>547304
Таки вы слышали за кванты? Оно или целиком поглощается или пролетает мимо во всех отношениях.
Аноним 12/10/21 Втр 11:16:50 54730868
>>547301
смотря в какую сторону расфокусировка, если мышцу расслабляешь - конечно экономит
Аноним 12/10/21 Втр 11:23:55 54730969
>>547287
О, аккреционный диск вокруг гигантского куска железа, нихуя себе.
Аноним 12/10/21 Втр 11:27:45 54731070
>>547306
Как это опровергает то, что я написал? Пирацетамчика захавай, потом отвечай.
Аноним 12/10/21 Втр 11:34:12 54731271
>>547310
Никак не опровергает. Пирацетам дорого. Дешевле накатить.
Аноним 12/10/21 Втр 11:37:34 54731372
>>547312
>Дешевле накатить.
Ещё и эффект реальный, одни плюсы.
Аноним 12/10/21 Втр 12:20:49 54731473
>>547153
Там дохуя разницы, ты СВЧ и ТГц пропустил. Они тоже поглощаются.
Аноним 12/10/21 Втр 12:22:25 54731574
>>547163
Ну так ты получишь, что минимальный размер - это примерно длина волны.
Аноним 12/10/21 Втр 12:46:18 54731675
>>547315
c двуя дисками и электроникой супротив пенетрирующего с интенсивностью скорости света реальность фотона? хуй за щеку получит этот экспериментатор (около 18 см) а не примерно длину (((волны)))
Аноним 12/10/21 Втр 13:42:07 54731776
>>547304
Если построить график поглощения в зависимости от длины волны, он будет не как плавная линия, а с вполне ярковыраженной, хоть и "скруглённой" ступенькой, так понятно?
Аноним 12/10/21 Втр 13:45:33 54731977
>>547317
>"скруглённой"
природа явления?
Аноним 12/10/21 Втр 13:47:32 54732078
>>547319
Квантовая вероятностная, ессесно.
Аноним 12/10/21 Втр 14:00:00 54732179
image.png 2055Кб, 900x995
900x995
>>547320
Но квант же резкий, как понос дерзкий. Откуда там плавность. У идеального меандра фронт взмывает с бесконечной скоростью.
Аноним 12/10/21 Втр 14:26:20 54732480
>>547321
>У идеального меандра
Не существует в реальности, только в твоей маняфантазии.
Аноним 12/10/21 Втр 14:29:11 54732581
>>547324
т.е. квант на самом деле непрерывный, прост оче гладкий на значительном удалении от пика?
Аноним 12/10/21 Втр 15:48:01 54732682
>>547325
Естественно, любое взаимодействия уходит в бесконечность, и ослабляется по какому-то закону
>Правда ослабление не по обратноквадратичному закону например хуярит нахуй теорию поля, и приходится вводить всякую хуйню типа виртуальных частиц-переносчиков
>Правда распространяется со скоростью света, и как там будет у фотона непонятно, с одной стороны фотон и сам летит со скоростью света, с другой и взаимодействия уходящее в бесконечность тоже со скоростью света, и по факту экспериментов фотон прилетает со 100%-й вероятностью в одно и то же места куда должен, а не во все направления, но тут классическое среднее квантовой механики, в итоге всё получается как и должно быть
Аноним 12/10/21 Втр 16:43:40 54732883
>>547326
но как же тогда поляризация уже поляризованного света не завершатся абсолютным затемнением если угол не перпендикуляр
Аноним 12/10/21 Втр 16:48:30 54732984
>>547328
Ну так благодаря эффектам квантовой механики, естественно.
Аноним 12/10/21 Втр 17:59:48 54733485
Посоветуйте книгу чтобы вкотиться в математику на старости лет. Типа как "Математика для взрослых" англичанина с незапоминаемой фамилией, только сложнее и подробнее. Хочу восстановить знания школьного курса алгебры и понюхать мат. анализ.
Аноним 12/10/21 Втр 18:50:13 54733786
Короче очередной парадокс, наблюдал в очередной раз за горением дров
Когда дрова прогорают и остаётся уголь, то его масса пиздей маленькая, он весь пористый и лёгкий
Но при этом по ощущениям угли дают пиздец много жара.
Казалось бы ничего такого, увеличилась из-за пористости скорость сгорания, но учитывая это и плюс их маленькую массу, они просто за секунды должны были выгорать, ведь они и есть топливо
А они "горят" пиздец как долго, при этом всё это время продолжая также давать дофига жара
Очень непонятная хрень, я когда маленький был думал что они кислород сжигают внутри себя, будучи горячими и пропуская его через свои пористые каналы, потом только узнал что кислород сам не горит и топливом не является.
Аноним 12/10/21 Втр 19:00:40 54733887
>>547337
>Но при этом по ощущениям угли дают пиздец много жара.
Угли больше тепла в излучение дают, т.к. они черные. Возможно это создает ощущение, что они больше тепла производят.
Аноним 12/10/21 Втр 20:01:12 54734088
>>547338
Ну ебать а дрова во что тогда дают больше тепла? Огонь горит и тоже излучает, а если бы угли дохуя хорошо излучали они бы просто остыли нахуй, у них же массы мало, ну или сгорели бы один хуй почти мнгновенно, опять же из-за того что массы мало.
И один хуй система же замкнутая, раз они такие горячие то через них проходит дохера воздуха, потому что он от контакта расширяется и улетает вверх по трубе, там постоянная тяга, т.е. они и воздуху полюбому много тепла передают, как ни крути.
А они и массы имеют хуй да нихуя, и пусть и в излучение дают много тепла, но и лежат долго не прогарая за секунды, и от постоянного обдува воздухом не остывают.

Но самая суть что в них просто пиздец насколько мало массы, допустим принёс кг 5 дров, они буквально быстренько прогорели за 30 минут, там тех углей осталось грамм 400 от силы, и они потом ещё те же пол часа лежат давая дохера жара свиду и воздуху его 100% много передавая.
Ну вот не поверю что они в 10 раз меньше тепла начинают производить, учитывая что имея в 10 раз меньшую массу лежат и не сгорают столько же по времени.
Аноним 12/10/21 Втр 21:02:06 54734289
2222.jpg 109Кб, 918x510
918x510
Какая глубина воды была в месте падения Чиксулуба, и на какую глубину метеорит пробил земную кору? Не нашёл такой информации.
Аноним 13/10/21 Срд 00:19:02 54734390
Антуан, поясни за фотоны. Может быть такое, что ваша вселенная вдоль и поперёк состоит из фотонов. И энергия переходит от одного фотона до другого, что нами воспринимается как движение фотона, а на самом деле, есть включенные и выключенные фотоны. Я понимаю, что я скорее всего просвещенные поржут с меня. Но поясни, где ломается эта теория.
Аноним 13/10/21 Срд 01:08:41 54734491
>>547343
>Но поясни, где ломается эта теория.
Вот возьми и попробуй сам какое-то явление объяснить своей теорией, потос следующее, потом следующее
А копаться в твоей шизе никому нахуй не надо, мало ли что ты там у себя в голове выдумал.
Аноним 13/10/21 Срд 07:04:56 54734792
УЧЕНЫЕ: АПОФИС ОПАСИН ОН МОЖЫТ УДАРИТЬ В ЗЕМЛЯШКУ
@
АБСОЛЮТНО ЧЕРНОЕ ТЕЛО В НЕСКОЛЬКИХ АСТРОНОМИЧЕСКИХ ЕДИНИЦАХ НА ТРАЕКТОРИИ СТОЛКНОВЕНИЯ С ЗЕМЛЁЙ: ТЫ МИНЯ НЕ УВИДИШЬ ПОКА Я НЕ ЕБНУСЬ В ЗЕМЛЯШКУ В ГОВНЕ МОЧЕНЫЙ ГЫЫЫЫ
Аноним 13/10/21 Срд 08:20:18 54734893
>>547347
Да мы и разноцветные метеориты хуй увидим. Вон как челябинский.
И увидев хуй сможем предотвратить столкновение.
Аноним 13/10/21 Срд 09:53:35 54735194
>>547343
Таблетки лучше прими, долбоеб.
Аноним 13/10/21 Срд 10:13:41 54735295
image.png 575Кб, 720x405
720x405
>>547343
не может, потому что фотон и есть минимальный кусок электрическо-магнетического потенциала. "на самом деле", если в современной физике такое вообще имеет смысл, он существует как эхо схлопнувшейся волновой функции. фотон - это вечное вчера.
Аноним 13/10/21 Срд 11:58:52 54735396
>>547343
Как бы сказать... Ты сейчас пальцем в небо попал, всё так и происходит.

Луркайте квантование электромагнитного поля и смотрите, как происходит разложение поля на осцилляторы.
Аноним 13/10/21 Срд 12:00:30 54735497
image.png 2569Кб, 1100x825
1100x825
Аноним 13/10/21 Срд 12:02:16 54735598
Фотон лол.png 40Кб, 806x385
806x385
Только не луркайте в википедии, там нихуя по этой теме нет.
Аноним 13/10/21 Срд 12:11:27 54735699
>>547355
>луркайте в википедии
унести это говно пожалуйста отсюда
Аноним 13/10/21 Срд 18:41:20 547363100
>>547120 (OP)
Поясните, почему у двух параллельных пружин с разной жесткостью одинаковое удлинение?
Аноним 13/10/21 Срд 23:38:08 547367101
>>547363
Потому что их так растянули. Вообще это не обязательно.
Аноним 13/10/21 Срд 23:50:57 547368102
vmeste245.jpg 19Кб, 470x234
470x234
>>547367
Я вот про подобные картинки. С последовательным понятно, но не понятно откуда для параллельного соединения удлинения должны быть равны. Как иногда пишут, что параллельное соединение пружин эквивалентно параллельному соединению конденсаторов, но для конденсаторов очевидно равенство напряжений в плечах, откуда берется равенство удлинении пружин мне не очевидно.
Аноним 13/10/21 Срд 23:57:37 547369103
Допустим я еду на платформе двтжущийся со скоростью 200км/ч
И кидаю мячик со скоростью 20км/ч в том же направлении что и движется платформа
Если я просто кидаю мячик со скоростью 20км/ч, то у него энергия условные 400
Если он просто с платформы падает со скоростью 200км/ч, то у него энергия 40000
А если я кидаю его с платформы, то у него энергия 48400
Ебать, я что, становлюсь в 8400/400=21 раз сильнее двигаясь на большой скорости?
Ну допустим я как-то отбирал энергию у платформы бросая мячь с неё, так если я сам максимум могу бросить с энергией 400, то передавая от платформы мячу энергию 8000 у меня бы просто нахуй руки сломались бы.
Аноним 14/10/21 Чтв 00:00:16 547370104
>>547368
Потому что тут упрощённый рисунок неправильный, очевидно что в жизни тут оно перкосится при таких точках опоры и шарнирах в таких местах, тут надо рисовать или полозья или систему рычагов с которыми общая соединительная планка может перемещаться исключительно ровно.
Аноним 14/10/21 Чтв 00:11:08 547371105
>>547245
>Тепловое излучение в первую очередь равновесное
Ты про то что та поверхность откуда я собирал излучение быстро охладится?
Или про то что была площадь 1 метр температурой 30градусов, я сконцентрировал до 10 на 10см, т.е. в 100 раз больше интенсивность, но излучение растёт как 4-я степень температуры, получается, 100^(1/4)= я всего примерно в 3 раза смогу температуру максимум поднять? ну пиздец
да, печально
А не, стоп, это ж в кельвинах, получается 303•3=636 градусов, вполне неплохо.
А там и теплоноситель можно гонять
Аноним 14/10/21 Чтв 00:11:21 547372106
>>547370
А если взять два кубика со стороной а, с разной упругостью, поставить их вплотную друг к другу и сверху положить кирпич с основанием со сторонами а и 2а, как деформируются кубики?
Аноним 14/10/21 Чтв 00:14:25 547373107
>>547372
Тоже будет перекос
Аноним 14/10/21 Чтв 00:15:36 547374108
А хотя не, походу не будет
Аноним 14/10/21 Чтв 00:26:51 547375109
Аноним 14/10/21 Чтв 01:38:42 547376110
>>547375
Потому что такое равномерное тело, будучи опёртое только на 1/2 своей площади практические само стоит и не падает, буквально 0.001% усилия поддерживающего хватит чтобы не падало

Но я чёт подумал что кубы только целиком могут сжиматься, не теряя перпендикулярность плоскости верхней, а если они типа как реальные упругие тела, то всё-таки походу будет перекос.
Аноним 14/10/21 Чтв 02:07:55 547377111
>>547120 (OP)
Поясните за сверхтекучесть. За счёт чего она течёт вверх и вообще о чём это говорит вообще...
Аноним 14/10/21 Чтв 02:26:45 547378112
>>547377
Мб лучше хотябы статью с таким названием в википедии хотябы прочтёшь?
Аноним 14/10/21 Чтв 03:10:26 547379113
>>547378
Я сюда пришёл за ответами.
Аноним 14/10/21 Чтв 08:19:13 547387114
>>547376
Чет хуита какая-то выходит, с конденсаторами есть четкое условие равенства напряжений в плечах, иначе между ними потечет ток, в случае с пружинами должно быть какое-то подобное условие. Какое?
Аноним 14/10/21 Чтв 11:47:41 547390115
>>547340
>дрова во что тогда дают больше тепла?
В разрушение структуры.
Аноним 14/10/21 Чтв 11:55:44 547391116
>>547369
Ты не бросишь так мяч из-за сопротивления воздуха, вот и всё. Иначе бы да, руки сломал.
Аноним 14/10/21 Чтв 12:12:01 547392117
>>547377
Поясняю: течёт вверх засчёт сверхтекучести, а вообще это говорит о том, что сверхтекучесть круче текучести. Сила связи между молекулами больше, чем сила тяготения.
>>547387
Почему ты принципы электротехники переносишь на механику?
Аноним 14/10/21 Чтв 13:23:42 547395118
В диайваче молчат, а больше мне некуда и пойти.

Чем гасить концентрированную серную, азотную, соляную кислоту или их смеси (чистка металла) так, чтобы результат можно было спокойно слить в ближайшую канаву или раковину ("продукты реакции безопасны для окружающей среды"), а реакция могла проводиться в закрытом помещении ("не была вредной для проводящего и не создавала дыма/пара, не требовала вытяжного шкафа")? Пробовал едким натром - получается адовая дымовуха как при пожаре.

Где купить этиловый спирт процентов 90 хотя бы, для горелок, обезжиривания, спасения утопающей электроники и т. д. в количестве 1-5л. Можно "технический", пить я его не планирую. В мосхимторге его внезапно не оказалось. Как-то в 2021 брагу перегонять ради этого - детский сад...
Аноним 14/10/21 Чтв 13:25:58 547396119
>>547387
Напряжение это как сила, а вот заряд на конденсах это именно деформация. Ты там размерности то хотя бы посмотри, у тебя и сила, и напряжение являются производными энергии по координате.
Аноним 14/10/21 Чтв 13:47:40 547397120
Существует ли синоним для термина "конденсат Бозе-Эйнштейна"?

Нужно использовать это понятие в НФ-книге, но там действие происходит в альтернативной реальности, отчего упоминание реальных имен из нашего мира не прокатит.
Аноним 14/10/21 Чтв 13:55:07 547398121
>>547397
КХБ – конденсат холодных бозонов. Пойдет?
Аноним 14/10/21 Чтв 14:15:26 547400122
>>547398
Бозоны от имени Бозе.
Вообще бы ему подошел бы просто квантовый конденсат, поскольку фермионы такого не образуют. Но тут главное не спутать с конденсатом квантового поля.
Аноним 14/10/21 Чтв 14:20:33 547401123
>>547391
Хуета твой ответ блять, нахуй ты вообще это высрал?
Аноним 14/10/21 Чтв 14:39:30 547403124
>>547401
Чтобы тебе было неприятно, немощная чмонька.
Аноним 14/10/21 Чтв 14:46:19 547404125
0.PNG 192Кб, 1528x580
1528x580
>>547395
Для технических целей тебе подойдет изопропиловый спирт - изопропанол (он не опасен как метиловый - погугли), можно купить на озоне от 500 мл или в химторгах, в магазинах радиодеталей не покупай - бешенные накрутки.
Аноним 14/10/21 Чтв 14:47:21 547405126
>>547400
"Ультрахолодный квантовый конденсат" - может так будет норм
Аноним 14/10/21 Чтв 14:58:27 547406127
>>547404
Да я конкретно этиловый хотел. Насколько он вообще подходит для обезжиривания и "вытягивания" воды из залитых микросхем? И что за хуета с этиловым спиртом - концентрированную серную кислоту купить можно, а спирт обычный - нельзя?
Аноним 14/10/21 Чтв 15:40:16 547409128
image 161Кб, 767x750
767x750
Я русский из Твери, вся родня славянские балтиды. Где мне искать жену с наиболее отличной генетикой? В африке? Может среди индейцев Амрики или Австралии? Хочу дать уникальное потомство, диверсифицировать так сказать генофонд нации
Аноним 14/10/21 Чтв 16:23:50 547412129
>>547397
Конденсат квантовых симметронов (фишка бозонов в симметричной волновой функции, по отношению к перестановке, это главное для них, это задает все свойства этого конденсата, симмтероны и антисимметроны вполне себе нормальные заменители слов бозоны и фермионы).
Аноним 14/10/21 Чтв 16:37:49 547414130
1634218667529.jpg 106Кб, 886x591
886x591
>>547409
> Я русский из Твери, вся родня славянские балтиды.
14/10/21 Чтв 16:58:01 547415131
>>547414
Пидорашка, брысь
Аноним 14/10/21 Чтв 17:59:32 547416132
>>547406
Дак все хотят этиловый, но мы живем в России - стране запретов, бессмысленных и беспощадных, водочные кабанчики наверное чего-то боятся, хз. Я изопропиловым отмывал платки от спиртоканифоли, очищал японские светодиодиоды, в их датащитах так и написано - чистить только изопропиловым спиртом По поводу вытягивания воды спроси в /ra.
Аноним 14/10/21 Чтв 18:03:47 547417133
>>547396
> заряд на конденсах это именно деформация
Но для параллельно соединенных конденсаторов нет требования равенства их зарядов.
>>547392
>Почему ты принципы электротехники переносишь на механику?
Потому что пока искал ответ на вопрос почему деформации равны натыкался на аналогию с конденсаторами.
Понятно, что mg=-k1x1-k2x2, но откуда следует x1=x2?
Аноним 14/10/21 Чтв 18:42:50 547419134
>>547417
>Но для параллельно соединенных конденсаторов нет требования равенства их зарядов.
Для паралельно соединенных пружин нет правила равенства их деформаций, пиздец блять
Аноним 14/10/21 Чтв 18:48:09 547420135
Новые научные теории возникают из-за необходимости объяснить явления, которые находятся вне "сферы досягаемости" современных моделей. Так вот, как в теории струн/теории петлевой квантовой гравитации описаны явления тёмной энергии и тёмной материи? Современные физики больше склоняются к мнению, что последняя - действительно некие массивные слабовзаимодействующие частицы или что проблема "скрытой массы" связана с иными механизмами гравитационного "взаимодействия" (нечто сродни модифицированной ньютоновской динамике)? Почему ОТО даёт более верный ответ на вопрос значения энергии вакуума, нежели квантовомеханические теории?
Аноним 14/10/21 Чтв 19:18:53 547421136
>>547420
Необязательно, некоторым просто не нравятся общепринятые модели, и они пилят парралельно свои, но естественно они надеются что их модели смогут объяснить и то что не объясняют общепринятые, но это необязательно.
Аноним 14/10/21 Чтв 19:56:23 547425137
>>547120 (OP)
Существует великое множество суперстабильных химических соединений. Возможно ли, что в один прекрасный день вселенная упрется в тупик реакций, который не выйдет покинуть?
Аноним 14/10/21 Чтв 20:22:04 547426138
>>547425
Что ты имеешь ввиду под стабильными?
Аноним 14/10/21 Чтв 20:43:20 547427139
>>547416
>Дак все хотят этиловый
Почему?
Аноним 14/10/21 Чтв 20:45:44 547428140
Что такое кварки?
Аноним 14/10/21 Чтв 21:05:02 547430141
>>547420
>действительно некие массивные слабовзаимодействующие частицы
Это. Причем тут, вполне возможно, наклевывается частица-кандидат в составляющую темного вещества. https://nplus1.ru/news/2021/10/12/Sterile-neutrinos
Аноним 14/10/21 Чтв 21:07:15 547432142
>>547428
Элементарные частицы класса фермионов, из которых построены адроны - сильновзаимодействующие составные частицы, такие как протоны, нейтроны, пи-мезоны и т.п.
Аноним 14/10/21 Чтв 21:08:34 547433143
>>547432
И откуда учёные это знают?
Аноним 14/10/21 Чтв 21:12:53 547434144
>>547433
https://ru.wikipedia.org/wiki/%D0%93%D0%BB%D1%83%D0%B1%D0%BE%D0%BA%D0%BE_%D0%BD%D0%B5%D1%83%D0%BF%D1%80%D1%83%D0%B3%D0%BE%D0%B5_%D1%80%D0%B0%D1%81%D1%81%D0%B5%D1%8F%D0%BD%D0%B8%D0%B5
https://ru.wikipedia.org/wiki/T-%D0%BA%D0%B2%D0%B0%D1%80%D0%BA
https://ru.wikipedia.org/wiki/%D0%9A%D0%B2%D0%B0%D1%80%D0%BA-%D0%B3%D0%BB%D1%8E%D0%BE%D0%BD%D0%BD%D0%B0%D1%8F_%D0%BF%D0%BB%D0%B0%D0%B7%D0%BC%D0%B0#%D0%98%D0%B7%D1%83%D1%87%D0%B5%D0%BD%D0%B8%D0%B5_%D0%BA%D0%B2%D0%B0%D1%80%D0%BA-%D0%B3%D0%BB%D1%8E%D0%BE%D0%BD%D0%BD%D0%BE%D0%B9_%D0%BF%D0%BB%D0%B0%D0%B7%D0%BC%D1%8B
https://ru.wikipedia.org/wiki/%D0%9A%D0%B2%D0%B0%D1%80%D0%BA#%D0%94%D0%BE%D0%BA%D0%B0%D0%B7%D0%B0%D1%82%D0%B5%D0%BB%D1%8C%D1%81%D1%82%D0%B2%D0%B0_%D1%81%D1%83%D1%89%D0%B5%D1%81%D1%82%D0%B2%D0%BE%D0%B2%D0%B0%D0%BD%D0%B8%D1%8F_%D0%BA%D0%B2%D0%B0%D1%80%D0%BA%D0%BE%D0%B2
Аноним 14/10/21 Чтв 22:10:05 547435145
>>547427
Не знаю, постоянно спрашивают этиловый, вот даже тут аноним хочет этиловый, хотя для его цели и изопропиловый ок, наверное чтобы после работы и отдохнуть можно было - шутка. А мне и изопропиловый нормально за исключением того, что когда несколько часов работаешь с изопропанолом без вытяжки то правда начинает болеть голова, я сначала думал совпадение, а потом где-то прочитал что это норма - есть такой эффект от паров этого спирта, так что если нет рабочего места с вытяжкой открывать окно.
Аноним 15/10/21 Птн 00:15:18 547437146
>>547419
>Для паралельно соединенных пружин нет правила равенства их деформаций
А хули весь гугель говорит наоборот?
Аноним 15/10/21 Птн 00:16:06 547438147
>>547420
>Новые научные теории возникают
таки из-за необходимости получения гешефта за щет налогов с гоев
Аноним 15/10/21 Птн 00:30:50 547439148
>>547435
А по безопасному гашению кислот можешь что-нибудь подсказать?
Аноним 15/10/21 Птн 00:50:17 547440149
>>547437
>>547437
То что они там в задачах для восьмиклассников параллельны, не означает, что они в равновесном состоянии такими хотят быть)
Аноним 15/10/21 Птн 00:50:25 547441150
>>547439
Нужно полное обезвреживание кислот или всего лишь нейтрализации pH?
Для нейтрализации достаточно обычной соды или извести, но там будет много газа. Есть вариант с оксидом железа или оксидом магния. Подойдет фосфат натрия, но его нужно много чтоб использовать как буфер.
Аноним 15/10/21 Птн 01:46:36 547442151
По пространству...
ооо, а что если тёмная энергия это гравитационные волны?
по пространству расходится всё больше и больше гравмтациоонных волн, они рассеиваются, но не исчезают, а просто ослабляются но действуют как бы на большем пространстве, получается везде более растянутое пространство когда волна рассеится, и таких вылетающих и рассеивающихся волн становится всё больше и пространство всё более и более растягивается, тёмная энергия.
Аноним 15/10/21 Птн 01:59:13 547443152
>>547442
Для темной энергии нужно отрицательное давление и постоянство, чтоб быть масштабным фактором. У гравитационных волн строго нулевое давление, в добавок они четно мультипольные(квадруполь, октополь, и т.д), что сильно ограничивает их проявление.
Аноним 15/10/21 Птн 02:02:03 547445153
Вот говорят шимпанзе на 1-1.5% отличается геномом от человека
И типа разные люди могут отличаться друг от друга
Это разные методики подсчёта которые вообще разное считают?
Аноним 15/10/21 Птн 02:52:08 547449154
Почему млекопитающие не вызывают страха, будь то кошачьи или собачьи, обезьяны или еще че, млекопитающие (пока они не начнут выказывать агрессивного поведения) будут казаться "безобидными"

Зато пауки, какие-то насекомые, здоровые ящерицы и крокодилы всякие, змеи, зубастые рыбы и все такое прочее вызывает первобытный страх хоть у младенца. Почему?
Аноним 15/10/21 Птн 03:08:18 547450155
>>547449
Гены не наши, не православные.
Аноним 15/10/21 Птн 06:02:27 547451156
>>547445
Это в среднем считается. Типа так: берём тысячу людей и сотню шимпанзе, сравниваем каждого шимпанзе с каждым человеком, считаем среднее арифметическое.
Аноним 15/10/21 Птн 06:11:40 547452157
>>547449
Это не совсем так.
1. Младенцы вообще не в состоянии видеть нормально, так что тут вопрос бессмысленнен.
2. Дети постарше боятся всего незнакомого, других людей, собак, кошек, еду, которую они до этого не ели, и т.д.
3. Взрослым неприятны пауки и змеи по дефолту, плюс животные крупнее них. Это обусловлено эволюционно.
4. Ещё люди боятся темноты, это тоже обусловлено эволюционно. Специализирующийся на приматах хищник один - леопард, а он ночной. И один хрен, зрение у нас под ночь не заточено.

Все перечисленные "инстинкты" довольно слабые, и у некоторых людей отсутствуют, к тому же их можно заглушить.
Аноним 15/10/21 Птн 08:07:42 547455158
Последний король ВБ,который хоть что то решал285888
Аноним 15/10/21 Птн 08:38:22 547456159
Как появились кроманьонцы?
Аноним 15/10/21 Птн 09:24:43 547457160
>>547441
Нужно чтоб не травануться, пока занимаешься этим в закрытом гараже, и не отравить местную землю, флору и фауну, когда сливаешь продукты реакции в канаву.
Аноним 15/10/21 Птн 10:13:23 547458161
>>547442
Темная энергия однородно заполняет пространство и обладает постоянной плотностью энергии, т.е. не меняется со временем. Гравитационные волны с этим вообще никак не могут быть связаны, это в принципе другое явление.
>>547449
>Зато пауки, какие-то насекомые, здоровые ящерицы и крокодилы всякие, змеи, зубастые рыбы и все такое прочее вызывает первобытный страх
У меня не вызывает. У любителей всякой экзотической живности тоже.
>>547456
Кроманьонцы - это сапиенсы, пришедшие в Европу из Африки. Сапиенсы, т.е. мы, сформировались 200 тыс. лет назад в Восточной Африке. "Как" - вопрос конечно непраздный, но очевидно, что путем биологической эволюции, т.е. постепенно так, не спеша. За 2 млн лет от ранних Homo докатились вот до сапиенсов.
Аноним 15/10/21 Птн 14:17:30 547464162
>>547457
Оксид магния лучше все подойдет.
Его легко достать, не дает газов и отходы можно сливать канализацию. Соли магния плохо растворимы в воде.
Аноним 15/10/21 Птн 15:18:30 547465163
>>547464
Да соды обычной хватит.
Аноним 15/10/21 Птн 15:38:52 547466164
>>547465
От соды слишком много газа будет, на ебло все выплеснется.
Аноним 15/10/21 Птн 16:08:34 547467165
>>547458
>мы, сформировались 200 тыс. лет назад в Восточной Африке
Учитывая что была скрещиваемость с неандертальцами и денисовцами, вышедшими из африки 700т. лет назад, не уверен от одних и тех же произошли денисовцы и неандертальцы, но отделять какой-то период "200т. лет назад" не вижу смысла.
Также встречал и другие цифры, про волны выхода из африки других волн, а может и этих же от которых пошли неандертальцы, аж почти в 2 раза больше, 1300т. лет назад.
Аноним 15/10/21 Птн 17:05:33 547474166
slide11.jpg 98Кб, 960x720
960x720
>>547337
Тупо больше тепла они и выделяет.

Особенно учитывая, что дрова часто НЕ сухие и часть тепла тратится на испарение влаги. Да и горение дров - это скорей горение газообразных продуктов пиролиза древесины - дающих высокое пламя, но таким образом размазывающие тепловыделение по объему.
А уголь - это пористый углерод с примесью минеральных остатков.

(Собственно поэтому в кузнечые/металургические горны, со времен медного века - засыпали уголь, а не дрова закладывали.)
Аноним 15/10/21 Птн 18:31:00 547476167
20211016002743.jpg 219Кб, 654x614
654x614
>>547467
>вышедшими из африки 700т.
Денисовцы сформировались 250 тыс. лет назад, а неандертальцы - 130 тыс. лет назад. См. пик.
>не вижу смысла.
Смысл есть, т.к. это время формирования нашего вида, к которому неандертальцы и денисовцы, будучи отдельными видами Homo, имеют весьма опосредованное отношение. Они всего лишь 30 тыс. лет назад дали нашему виду немножко своей ДНК, часть из которой за это время оказалась выбракована отбором и до сегодняшнего времени дошли какие-то жалкие остатки, которые даже ни на что не влияют в плане функциональности.
Аноним 15/10/21 Птн 18:47:17 547477168
>>547148
Это же колебания, а не кусок вещества. Если лететь со скорость 365 м/c и орать - скорость звука не станет 700 м/с.
Аноним 15/10/21 Птн 21:07:43 547483169
>>547267
Возможно ты обосрался и имел ввиду не астрофизику, а фундаменталью или еще какую, потому что с точки зрения астрофизика ЧД - это гравитирующий объект не проявляющий признаков поверхности, не больше не меньше. Что там внутри ЧД и что это вообще такое астрофизикам до лампочки, у них наука самая практическая - наблюдательная. И таких объектов астрофизики уже вагон и маленькую тележку открыли. Ну и соломка подстелена на самом деле со стороны фундаменталистов: для них ЧД - это математический объект,а то, что там наблюдают астрофизики они называют кандидатами в ЧД, причем кандидатами они останутся навсегда, потому что доказать наличие горизонта у объекта практически нереально
Аноним 15/10/21 Птн 21:17:30 547484170
>>547464
Спасибо. Реакция с концентированными HCL, H2SO4, HNO3 идет полностью, до нейтрализации ph? Алсо, в интернете советуют гидроксид кальция - хуже?
>>547464
>Его легко достать
Стоит недешево. Или его можно в виде каких-нибудь удобрений и стройматериалов нарыть?
Аноним 15/10/21 Птн 22:10:37 547486171
>>547484
Оксид магния дает очень слабое основание, так что pH будет болтаться около 7-8. Гидроксид кальция более сильное основание. В добавок сульфат кальция хуже растворяется в воде, чем сульфат магния.
Поищи в стройматериалах или добавок для цементов. Там он может быть не чистым, а в составе смесей.
15/10/21 Птн 22:53:54 547488172
>>547487
>ненавидящая наше государство
Что за ваше государство? Ученые всегда были и есть леваки, и вообще граждане галактики, так что катись со своим половым примитивным обезяньим батхертом к хуям.
Аноним 15/10/21 Птн 23:06:24 547489173
>>547476
Это когда он сформировался в том виде в котором он уже считался таким, а я про дату выхода из африки/отделения той группы из которой он сформировался.
В раз скрещиваемость осталась, то можно эту самую древнюю точку разделения брать и считать что это всё равно несмотря вот на такое дальнее отделение был один вид.
Просто другой подвид.
Аноним 15/10/21 Птн 23:08:40 547490174
>>547476
>которые даже ни на что не влияют в плане функциональности
>единственные цивилизации построены теми у кого есть примеси неандертальцев и диничевцев, африканцы же сидят в африке как и сидели 1млн лет назад.
Хех мда
Аноним 16/10/21 Суб 03:47:32 547493175
Аноним 16/10/21 Суб 06:54:44 547494176
>>547490
У всяких чукч и североамериканских индейцев примеси неандертальцев тоже есть.
Построить цивилизацию им это не помогло.
Аноним 16/10/21 Суб 07:34:32 547499177
А что если в качестве источника дармовой энергии использовать аннигиляцию? Взять какой-то природный бета-излучатель и бомбардировать его позитронами, они будут аннигилировать и гамма кванты при помощи фотоэффекта переводить в электричество.
Аноним 16/10/21 Суб 08:28:04 547500178
x066b774c.jpg 34Кб, 500x252
500x252
Аноним 16/10/21 Суб 10:20:16 547503179
>>547489
>считать что это всё равно несмотря вот на такое дальнее отделение был один вид.
Ну ты можешь считать все что угодно, только науке это все равно. Это по факту разные виды и объединение их в один вид невозможно, т.к. ни на чем не основано. Иди учи критерии вида. Чтобы кого-то считать одним видом, они должны между собой успешно скрещиваться и давать плодовитое потомство. Потомство от смешения сапиенсов с неандертальцами и денисовцами было в подавляющем большинстве случаев неплодовитым, плодовитыми оказывались лишь немногие. Т.е. отличий между этими популяциями накопилось уже достаточно, чтобы у них не получалось почти плодовитого потомства. А это значит, что мы имеем дело с разными видами.
>Просто другой подвид.
Другой подвид - это раса в рамках одного вида. Например, современные расы можно назвать подвидами вида Homo sapiens.
>>547490
И что ты этим хотел сказать, дебич? Ты каким-то магическим, одному тебе известным способом увязал продвинутость в плане цивилизации с неандертальско-денисовской примесью? Так примерно почувствовал, да? Лол блядь, какой же даун.

Открою тебе секрет: неандертальцы были довольно туповатыми и примитивными, по крайней мере точно тупее нас. Об этом говорит хотя бы тот факт, что мы есть, а их как-то не особо видно. Кроме того, их материальная культура (орудия труда, погребения, искусство, которого у них почти не было) по сравнению с культурой сапиенсов очень бедная и примитивная. Это были такие приземистые крайне массивные, ростом метр с кепкой, очень агрессивные троглодиты со страшными ебальниками как у Валуева, которые не чужды были каннибализма сапиенсы, впрочем, тоже за этим были замечены, но у неандертальцев этого прям дохуя. Про денисовцев еще мало известно, но они тоже вряд ли были какими-то гениями - их-то тоже нету. Поэтому на какую-то "прогрессивность" в плане построения цивилизации, на высокий интеллект примесь неандертальцев и денисовцев точно влиять не может. Так что ты сейчас жидко обосрался себе в штаны.
Аноним 16/10/21 Суб 10:22:14 547504180
>>547494
И что? Если какие-то птицы не могут научиться летать и разбиваются, выпадая из гнезда, это же не значит, что крылья не дают возможность летать.
Аноним 16/10/21 Суб 10:23:24 547505181
>>547503
>Потомство от смешения сапиенсов с неандертальцами и денисовцами было в подавляющем большинстве случаев неплодовитым, плодовитыми оказывались лишь немногие.
Пруфы, шиз.
Аноним 16/10/21 Суб 13:44:58 547506182
>>547504
И то, что чёрные цивилизации не строили не из-за генов.
Аноним 16/10/21 Суб 15:57:34 547510183
>>547505
>На основе изученного, пиком можно считать период порядка 60-40 тысяч лет назад, когда одновременно жили в Европе и Западной Азии – неандертальцы, в Восточной Азии – денисовцы, а в Африке – сапиенсы. Это были отдельные виды людей, которые отличались друг от друга, но при этом могли скрещиваться и давать потомство – иногда плодовитое.
>иногда плодовитое
http://erazvitie.org/article/evolutsiya_cheloveka
Аноним 16/10/21 Суб 16:38:31 547513184
>>547503
>Это по факту разные виды и объединение их в один вид невозможно
Чел блять, названия видов посмотри
Аноним 16/10/21 Суб 18:29:40 547517185
20211017002708.jpg 500Кб, 1034x920
1034x920
>>547513
>Чел
Пиздец, как же заебали тупые пориджи.
>названия видов посмотри
Ну вот и посмотри, малолетний дегенерат. См. пик.
Аноним 16/10/21 Суб 19:30:53 547518186
>>547517
Даун, что за картинки для даунов ты смотришь.
Аноним 16/10/21 Суб 19:38:29 547519187
>>547503
>Другой подвид - это раса в рамках одного вида. Например, современные расы можно назвать подвидами вида Homo sapiens.
Да вы заебали.
Раса не является строгим таксономическим рангом, у неё нет определения, потому что так и не получилось ничего внятного придумать. Соответственно, никаких строгих границ между расами нет, а сам термин содержит мало смысла.
Для подвидов нужна географическая или сексуальная изоляция, которой в реальности у вида homo sapiens никогда не было. Все близлежащие племена/общины постоянно скрещивались друг с другом, а географические изоляции, хоть и существовали время от времени, они всегда прерывались. Даже в Северную Америку было несколько волн миграции.

Если тебе прямо вот вынь да положь подвиды человека — то есть сентинельцы и им подобные племена. Всё остальное человечество активно скрещивается и является единой популяцией, и являлось ей всегда с самого возникновения вида.
Аноним 16/10/21 Суб 20:25:40 547523188
Аноним 17/10/21 Вск 03:18:24 547539189
Блядь я уже два дня безрезультатно гуглю, помогите с вопросом по сопромату.

Какой метод/алгоритм используется во всех этих программах на телефоне для расчета балок и их эпюр, в том числе и статически-неопределимых??? Я нашёл тонну инфу и видосов по эпюрам моментов и поперечных сил, но ВСЕ они аналитические, и ни одного объясняющего как это сделать чисельно в на питоне, например. Халп, буду бампать до смерти или до ответа по теме! Нет, не метод конечных элементов.
Аноним 17/10/21 Вск 06:41:30 547542190
>>547523
Бремя доказательств лежит на утверждающем, а не на отрицающем.
Аноним 17/10/21 Вск 06:48:25 547543191
>>547519
>15% генетического разнообразия человека приходится зависит от расы
>блевачёк пиздит, что рас нет и вообще человечество одна популяция
Долбаёб? Долбаёб.
Аноним 17/10/21 Вск 06:55:16 547544192
>>547543
А ты в курсе, что у всех собак, например, тоже одна популяция? Можешь называть себя породой обезьяны, никто не против.
Аноним 17/10/21 Вск 07:09:46 547547193
>>547544
>А ты в курсе, что у всех собак, например, тоже одна популяция?
Даже у тех, которых кинологи смешивают только внутри породы? Ври дальше.
>Можешь называть себя породой обезьяны, никто не против.
Подмена понятий? Как грубо.
Аноним 17/10/21 Вск 13:17:39 547558194
Нахуя человеку вообще углеводы, если на выходе всё равно СО2?
Т.е. что белки что углеводы проходят полный цикл до СО2, так какая разница
Типа цикл полный, но по разным цепочкам осуществляется и это важно?
Аноним 17/10/21 Вск 13:23:07 547559195
>>547542
>чёрные цивилизации не строили не из-за генов
Вот и доказывай утверждение.
Аноним 17/10/21 Вск 13:24:36 547560196
Аноним 17/10/21 Вск 13:35:08 547561197
>>547503
>увязал продвинутость в плане цивилизации с неандертальско-денисовской примесью?
А по-твоему она неочевидна пиздоглаз?
Аноним 17/10/21 Вск 13:38:23 547562198
>>547458
>Темная энергия однородно заполняет пространство и обладает постоянной плотностью энергии, т.е. не меняется со временем. Гравитационные волны с этим вообще никак не могут быть связаны, это в принципе другое явление.
Ты вкурсе как вообще это расширение измеряется, какие маркеры и какая вообще у таких маркеров допустимая погрешность?
Аноним 17/10/21 Вск 17:24:51 547563199
>>547561
"Очевидно" тебе может быть что угодно, хоть хуй во рту у твоей мамаши, даже если его там нет хотя это неправда, он там есть всегда. Вопрос только, какое отношение это имеет к науке. В данном случае твои маняочевидности есть не более чем причуды твоего магического мышления, пидорашка, только и всего. Т.е. это не имеет к науке никакого отношения.
>>547562
Антисетипетух, спок.
>>547553
>находим их икону Дробышевского
Кек, тупой блевак триггернулся. Открою тебе секрет, блевак: если какой-то ученый высказывает мнение, которое не вписывается в твою леволиберальную лгбт-повесточку, то из-за этого он не перестает быть ученым. Напротив, тем самым он показывает свою независимость от тоталитарной блевацкой маняидеологии, которая стремится вторгнуться в науку и полностью подмять ее под себя (лысенковщина 2.0). И это при том, что Дробышевский сам блевак, только не мейнстримный.
>но и за то, что эволюционно пидоров не должно быть
Как понять, что блевак пиздит? Он открывает рот. Дробышевский нигде не говорил, что пидоров эволюционно не должно быть. Он говорил о том, что гомосексуальность с точки зрения эволюции - это сбой, нарушение. Т.е. это такая ошибка природы. И это строго по факту.
>поэтом гомофобия оправдана
Что такое "гомофобия"? Вот если пидарасы вызывают у меня омерзение, я "гомофоб"? Хотя конечно же у этого растяжимого понятия нет никакого четкого и понятного определения, как и у любых других слов в левацком новоязе. Очевидно, "гомофоб" - это любой человек, который имеет позицию, не вписывающуюся в лгбт-повестку, а следовательно, очень не нравящийся блевацким сектантам. Я, к примеру, не собираюсь убивать пидоров, скармливать их собакам или как-то регулировать их сексуальную жизнь. Хотят ебаться в жопу - пусть ебутся наздоровье. Только вот хуй им, а не усыновление детей и однополые браки. Такой хуйни допускать нельзя. В понимании блеваков я наверное лютый гомофоб.
>глобального по-тепления с одной стороны не существует
Чушок-верунок в "страшное-ужасное антропогенное потепление", ты уже помолился швитой Грете Тунберг, отказался полностью от мяса, транспорта, одежды, стерилизовался? И да, Дробышевский опять-таки нигде не отрицал сам факт потепления. Он отрицает лишь бредни алармистов о том, что это потепление якобы вызвано деятельностью человека и что оно такое "страшное" и "ужасное".
Аноним 17/10/21 Вск 17:25:30 547564200
Аноним 17/10/21 Вск 18:05:35 547565201
>>547563
>то из-за этого он не перестает быть ученым.
Он перестает им быть, когда выходит за области своего профессионального знания. А это он делает постоянно. Та же гомосексуальность по многим причинам изучена мало и по понятным (не тебе) причинам изучать ее не очень интересно. Но я, так и быть, объясню: предположим Дробышевский прав (а это не факт, потому что на данный момент таких данных нет) и гомосексуальность - какой-то там генетический сбой и существует только благодаря генетическу дрейфу и в будущем возможно будем выведена из популяции, как вредная мутация. Что дальше? Все выводы из этого (как и из противоположного) в гуманитарной сфере (права человека) лежат за гранью биологии. Да, та самая натуралистическая ошибка (наделение моральными качествами естественные процессы) тут во своей красе. И очень жаль, что на эту херню ведутся приличные люди и начинают на эту тему спорить, доказывая, что гомосексуальность естественна и существует в дикой природе
>Что такое "гомофобия"?
Одна из разновидностей ксенофобии. И вот то, что ты тут высрал - это она самая и есть, в парижскую палату мер и весов прямо, даже комментировать не надо.
>Он отрицает лишь бредни алармистов о том, что это потепление якобы вызвано деятельностью человека
Вот очередной пример, как ученый вылезает за пределы своей компетенции и несет чушь. Чем он тут лучше Лайнса Полинга? А ведь даже нобелевки не получили.
>>и что оно такое "страшное" и "ужасное".
Ну да, подумаешь, дохуя людей помрет, не говоря уж про изменения экосистем и непредвиденные по этому поводу последсвия, никаких ужасов и не такое переживали. Как-то так он это рассказывает и это, честно говоря, звучит еще более чудовищно, чем тупо антопогенодиссиденство
Аноним 17/10/21 Вск 18:54:03 547567202
>>547565
>смачный подрыв блевака
Аноним 17/10/21 Вск 19:18:31 547568203
>>547563
Русский ванка-хуесос порвался на отличненько))
Аноним 17/10/21 Вск 19:19:24 547569204
Аноним 17/10/21 Вск 19:38:58 547570205
>>547569
Просто берут и в формулы подставляют значения
То что ты видишь, типа балка-хуярка, это нехуя не модель а просто способ ввода
Вот например вводишь ты балка 5 метров, оно не считает потом модель балки 5 метров, оно бля просто в формулу подставляет значение l=5
Выбираешь другой вид опоры, другую нагрузку, оно тоже нихуя не считает другую модель, оно просто выбирает другую формулу, типа выбрал жёсткая/скользящая заделка, оно поняло по какой формуле считать и в неё потом вбивает значения, распределённая/точечная нагрузка/момент крутящий/изгибающий тоже другую формулу выберет
Аноним 17/10/21 Вск 19:44:47 547571206
>>547570
>и в формулы подставляют значения
>другую формулу выберет
Шизик, нет никаких формул. Зачем ты отвечаешь, если не знаешь как решаются подобные задачи?
Аноним 17/10/21 Вск 19:54:28 547572207
>>547571
>нет формул
Ага долбаёб, молитву читают и боженька блять подсказывает ответ.
Аноним 17/10/21 Вск 19:57:06 547573208
>>547572
Порридж спок, не все в жизни так как учат тебя в тиктоке.

>>547539
Бамп.
Аноним 17/10/21 Вск 20:23:42 547574209
>>547568
Чучмек пиздоглазый, спок.
Аноним 17/10/21 Вск 21:46:12 547576210
Чем ограничена температура горения углерода?
Т.е. в атмосфере продукты горения будут расширяться и охлаждаться тем самым
А если в закрытом объёме с достаточным количеством кислорода?
Аноним 17/10/21 Вск 22:21:06 547577211
>>547576
В вкратце, при большой температуре диоксид и оксид углерода могут разрушатся, попутно блокирую радикалы и ингибируя реакцию. Меньше скорость реакции - меньше загорев и медленнее поднимается температура. При очень больших температурах и давлениях реакции может совсем остановится.
Аноним 18/10/21 Пнд 14:46:01 547594212
>>547573
Зумерок со второго курса институтика ты, а порридж почему-то он, лол.
Аноним 18/10/21 Пнд 14:49:11 547595213
>>547594
Пердикс спок, я шарагу закончил когда ты еще под стол пешком ходил. Подтяни матчасть, а потом пизди про формулы для постройки эпюр. Все свободен.

>>547539
Бамп.
Аноним 18/10/21 Пнд 15:18:35 547596214
Сколько процентов воспринимаемой человеком информации идет через зрение?
Аноним 18/10/21 Пнд 15:37:28 547598215
>>547596
Мозг не процессор, сенсорные органы не каналы связи, поэтому в процентах тебе никто не скажет. Но для приматов зрение - крайне важный источник, может быть даже самый важный.
Аноним 18/10/21 Пнд 15:38:02 547599216
Что такое скалярное поле и чем оно так уникально в отличие от других полей (векторного э/м и тензорного гравитационного)?
Поле Хиггса - скалярное, что в нем такого особенного?

Скаляр - просто число и больше ничего, значит ли это, что скалярное поле - это поле, кванты которого могут характеризоваться только голой величиной энергии и не имеют больше никаких характеристик?
Аноним 18/10/21 Пнд 15:54:06 547600217
Можно ли придумать идею которую другой произвольный разум принципиально осознать не способен?
Аноним 18/10/21 Пнд 16:14:10 547601218
>>547600
Множество, которое включает в себя все множества, элементами которых не являются сами эти множества. Справься своим разумом сначала с этим, а потом уже задавайся вопросами о чужих разумах.
Аноним 18/10/21 Пнд 17:07:36 547603219
>>547599
Насколько я понимаю эту концепцию: скалярное поле - это поле чистой энергии. Векторные, тензорное, спинорные компоненты обычных полей материи придают им различные специфические свойства (заряд, спин, цветной заряд, гиперзаряд и т.д.).
Квант скалярного поля же принципиально лишён всех свойств, кроме величины энергии.

Это совершенно непохоже ни на фемионную (вещество), ни на бозонную (калибровочные поля) материю, какая-то совсем новая форма самой экзистенции, "энергия без свойств".
Аноним 18/10/21 Пнд 17:44:03 547604220
Аноним 18/10/21 Пнд 17:49:01 547605221
>>547599
>Что такое скалярное поле
Поле, которое не имеет выделенного направления, очевидно.
Аноним 18/10/21 Пнд 17:49:03 547606222
>>547604
Ожидаемо, не справился
Аноним 18/10/21 Пнд 19:00:27 547608223
>>547600
Ты сам являешься произвольным разумом, значит тебе придется придумать идею, которую ты сам не в состоянии осознать.

Ты обратился по адресу, любой двачер и любая ТП такое может - твоя идея должна быть лишена смысла изначально, например будучи произвольным набором символов. Т.е. ее смысл невозможно вывести из написанного, просто потому, что в самой записи нужной идеи этого смысла нет - следовательно осознать ее можно только присвоив ей какой-то смысл, что будет ошибкой, ведь сама идея в том, что смысла ты не закладывал.
Аноним 19/10/21 Втр 01:58:17 547619224
Как Томасу Юнгу удалось в 1803 году сделать прорези в 400-800нм для своего опыта с интерференционным узором? Я хочу воспроизвести его.
Аноним 19/10/21 Втр 02:23:04 547620225
>>547619
Наверняка продрочил своей маленькой пипиркой...
Аноним 19/10/21 Втр 03:11:22 547625226
Аноним 19/10/21 Втр 09:32:11 547636227
сап такой вопрос я летом проходил тест на iq и решил сегодня перепройти считай спустя 2 месяцев.такой вопрос будет ли этот результат точным или все такие надо еще подождать?
Аноним 19/10/21 Втр 10:59:07 547642228
>>547619
Две щели это современный вариант опыта. Юнг пропускал солнечный свет через маленькое отверстие и полученый тонкий луч направлял на картонку которой разделял этот луч на две части. Насколько я понимаю тут идея что луч света должен быть ненамного больше толщины картонки
Аноним 19/10/21 Втр 11:34:45 547645229
>>547608
Ты пишешь какие-то глупости. Я хочу чтобы была нормальная идея, которую одни осознать способны, а другие нет. Как в том видео "альтернативная математика". Вот чтобы для этого разума все остальные были быдлом как будто специально
Аноним 19/10/21 Втр 12:11:15 547648230
Наконец спрошу - что произойдет, если измерить спин электрона 2 раза подряд?
Например имеем електрон в вакууме, его спин в суперпозициии, мы не знаем какой именно. Меряем, происходит декогеренция(верно?) и теперь мы знаем конкретный спин. Что произойдет, если мы померяем спин еще раз? И еще раз и еще - спин будет всегда равен изначально измерянному, или будет каждый раз произвольный?

А если мы померяем точно координату электрона, то его направление движения станет неопределенным, так? Т.е. узнавая какой-то параметр электрона точно, мы заставляем другой параметр становиться максимально неопределенным?
Аноним 19/10/21 Втр 12:15:33 547649231
>>547547
>Даже у тех, которых кинологи смешивают только внутри породы? Ври дальше.
Их искусственно ограничивают на какие-то сраных несколько поколений, это нещитово по эволюционным меркам.

От того, что в стране Х запрещали ебаться белым с чёрными, они не становятся разными популяциями.
Аноним 19/10/21 Втр 12:19:55 547650232
>>547563
Чем тебе так однополые браки не нравятся? Это ведь не твоё дело, кто с кем ебётся и ведёт совместное хозяйство - сам же сказал.
Аноним 19/10/21 Втр 12:22:58 547651233
>>547595
Чел, тебе нужен ответ - тебе ответили. Формулы там внутре. Аналитические. Хочешь численно - пиздуй в конечные элементы.
Аноним 19/10/21 Втр 12:24:38 547652234
Аноним 19/10/21 Втр 12:26:47 547653235
>>547619
Там не 400-800. Провести лезвием по стеклу, намазанному тонким слоем краски - хватит.
Аноним 19/10/21 Втр 12:27:30 547654236
Аноним 19/10/21 Втр 12:29:45 547655237
>>547648
Да, будет такой же. Идеальный измерительный прибор математически обозначается проекционным оператором.
Аноним 19/10/21 Втр 13:47:31 547660238
>>547648
>Т.е. узнавая какой-то параметр электрона точно, мы заставляем другой параметр становиться максимально неопределенным?
Только те параметры, у которых коммутатор с измеряемым ненулевой. Например измеряя координату по x ты не "портишь" импульсы по y и z.
Аноним 19/10/21 Втр 14:07:12 547662239
Поясните за многомировую интерпретацию эверетта.
Вот мы наблюдаем фотон, прохождящий через щель - соответственно запутываемся с ним и разделяемся на наблюдателя, с фотоном у щели 1, и наблюдателя с фотоном у щели 2.
С этим в принципе понятно.
Но что происходит, когда мы не наблюдаем фотон у щели, а видим интерференционную картину на экране?
Точне даже не так, если через щели пролетает фотон и попадает на какую-то из интерференционнных полос, и мы видим на экране эту точку, куда попал фотон - это значит, что экран по сути и есть детектор и вселенная расщепилась на бесконечно много вариантов, и мы наблюдаем тот из них, куда в нашей "вселенной" попал фотон? Если так, то чем это отличается от детектора у щели и картины с двумя полосами и двумя вселенными?
Или в случае интерференционнной картины расщепления не происходит?
Аноним 19/10/21 Втр 14:19:02 547665240
>>547645
Ну тогда тебе сначала нужно дать определение "осознание"
Аноним 19/10/21 Втр 14:32:41 547666241
>>547662
Происходит. При любом квантовом явлении происходит расщепление вселенной.

Не советую глубоко загоняться по интерпретациям, они особо не нужны.
Аноним 19/10/21 Втр 14:38:33 547667242
>>547666
Ну интересно же углубиться.
Вопрос тогда, почему, если детектор возле щели - то вселенная расщепиться на 2, а если интерференция на экране - то вселенная расщепиться на бесконечно много, в соответствии с количеством атомов экрана. Я явно что-то неправильно понимаю.
Аноним 19/10/21 Втр 15:48:54 547673243
>>547651
Не существует никакого набора аналитических формул для нахождения моментов и поперечных сил в сложных системах. Для каждой геометрии эти формулы выводятся аналитически. Я задал вопрос по программам расчёта, которые внезапно не считают интегралы для нахождения этих формул, а используют числовые методы. И эти методы не МКЭ потому что программы не для расчета конечных элементов ебаный в рот.

>>547539
Бамп.
Аноним 19/10/21 Втр 17:18:57 547681244
>>547667
Это докен доказал?

>>547673
>И эти методы не МКЭ потому что программы не для расчета конечных элементов ебаный в рот.

Чего? Про краевые условия слышал?
Аноним 19/10/21 Втр 17:20:53 547682245
image.png 76Кб, 200x200
200x200
>>547666
>При любом квантовом явлении происходит расщепление вселенной
Аноним 19/10/21 Втр 17:26:06 547683246
>>547681
Я не просто про краевые условия слышал, я для магистерской свой солвер для FEA писал. И прекрасно могу отличить программы с решателем на мкэ от других. Не лепи горбатого, просто забей и я подожду того, кто в курсе моего вопроса.
Аноним 19/10/21 Втр 17:33:25 547685247
>>547681
>Это докен доказал?
Да я хз.
Я понял, что расщепление происходит не всегда, а при декогеренции, когда среда запуталась с объетом, или детектор запутался с объектом.
И вот я подумал, экран с интерференционной картиной эквивалентен большому количеству маленьких детекторов, когда на какой-то из них попадает фотон - он запутывается с ним, и получается деление.
И вот, если "экран" у нас 100 на 100 - и интерференционная картина размазана по всему экрану - то получается деление на 10к вселенных.
Но если детекторы прямо у щелей - то получается всего 2 вселенные.
Может это конечно потому, что ставя детекторы у щелей мы уловим только те "варианты" фотона, которые "летели бы" достаточно прямо, чтобы образовать только 2 пятна на эакране.
Бля, все это конечно слишком умозрительно.
Аноним 19/10/21 Втр 17:36:23 547686248
>>547685
Выпей таблеток, шиз.

>>547683
Я, как и многие аноны тут, нихуя не понял в чём проблема использования мкэ. Или ты методом монте-карло решил интегралы считать?
Аноним 19/10/21 Втр 17:39:46 547688249
>>547673
> Я задал вопрос по программам расчёта, которые внезапно не считают интегралы для нахождения этих формул, а используют числовые методы. И эти методы не МКЭ потому что программы не для расчета конечных элементов ебаный в рот.
Ахаха, долбаёб, откуда ты знать можешь если ты нагуглить не смог информацию, ахахахаха
Аноним 19/10/21 Втр 17:56:34 547691250
>>547686
>Выпей таблеток, шиз.
Я пытаюсь понять многомировую интерпретацию. Очевидно, что я легко могу быть неправ и уж тем более я не пытаюсь никому ничего навязать.
Или ты считаешь шизой все, что связанно с многомировой интерпретаций?
Аноним 19/10/21 Втр 18:00:18 547692251
>>547667
Почти правильно, сначала при измерении, потом еще раз, когда уже измеренный фотон долетел до детектора. С измерением расщеплений получается в два раза больше, чем без измерений.
Аноним 19/10/21 Втр 18:13:58 547694252
>>547650
Пусть ебутся с кем хотят, но общество не обязано давать им никаких особых прав и привилегий, в частности вводить специально для них какую-то особую форму "брака" (однополый брак - это вообще оксюморон блядь, т.к. брак по определению является союзом мужчины и женщины) и давать состоящим в этом "браке" пидоркам усыновлять детей в свою пидорскую пару. Вон чурки тоже хотят, чтобы им легализовали многоженство. По логике блевацкой повесточки, продвигающей всю эту хуиту с однополыми браками и прочей дичью, нужно легализовать и многоженство. И так можно легализовать еще много какую хуйню, в т.ч. брак с животными, чтобы уважить все сраные обижающиеся маняменьшинства. Я считаю, что маняменьшинства должны идти на хуй со своими обижульками и требованием "особых прав", поэтому я против однополых браков и тому подобных вещей.
Аноним 19/10/21 Втр 18:17:56 547696253
>>547662
>Поясните за многомировую интерпретацию эверетта.
Бесполезная хуита, не предсказывающая ничего нового и к тому же ненаучная ввиду своего несоответствия критерию Поппера.
Аноним 19/10/21 Втр 18:22:56 547697254
>>547686
> Я, как и многие аноны тут, нихуя не понял в чём проблема использования мкэ
Проблема в том, что мой вопрос бы не в том, можно ли решить балку с мкэ. Мой вопрос был в другом.
Аноним 19/10/21 Втр 18:35:09 547698255
>>547697
У меня вопрос: когда ты хуй изо рта вынешь уже и нормально уже скажешь, что и зачем тебе надо.
Аноним 19/10/21 Втр 18:38:39 547699256
>>547691
Бор же сказал, что молодой человек - наркоман и попутал. Ты что не веришь Бору?
Аноним 19/10/21 Втр 19:05:49 547702257
>>547694
Ну так просто отмените налоги, и дотации которые идут женатому говну и их выблядкам, всё просто.
А то иш, какое-то говно плодится, которое даже не может само содержать своих выблядков, а на пидоров которые хотят состоять в браке что-то кукарекают.
Аноним 19/10/21 Втр 19:52:19 547708258
>>547698
Сейчас мать твою доебу она на пальцах объяснит сыну долбоебу.
Аноним 19/10/21 Втр 20:14:28 547710259
>>547702
>Ну так просто отмените налоги, и дотации которые идут женатому говну и их выблядкам, всё просто.
Семья - ячейка общества. Обществу выгодно поддерживать семьи. И да, конечно же речь идет о нормальных семьях. Пидорские парочки к семьям не относятся, т.к. они не могут даже потенциально ни рожать ни нормально воспитывать детей. Ну и социальная поддержка семей должна осуществляться разумно, т.е. деньги должны получать реально нуждающиеся, например семьи молодых ученых. Так, как это делается у нас сейчас, когда государство просто выделяет подачку и она размазывается по "семьям" всяких ебаных алкашей и прочих пидорах, которые эти деньги просто пробухивают и прожирают, а дети им нахуй не нужны, это конечно же полнейшая хуйня и неправильно. Систему нужно менять.
Аноним 19/10/21 Втр 21:53:57 547717260
>>547409
в москве концентрация красивых умных здоровых жена. самая большая в России и по относительным числам и по абсолютным. город дорогой по съему жилья - в итоге задерживаются более качественные приезжие. еще жирнее в калифорнии \ нью-йорке.

если тебе нужны самые стройные - то вьетнам. там 40млн тян и последнее место в мире по ожирению.

самые вичевые тян в Эсватини. можно вспоминать тверь. вообще нужнео смотреть какие у тебя фетиши(очки \грудь \ имплантанты \ тюремные тату\волосатый лобик итд)>>547409

ты делаешь акцент на генетическое качество. тогда надо смотреть на возраст 14 -36 нормально . после 36 повышенный риск даунов при рождении.. в целом в универах тян генетически качественнее чем в пту. особенно в лучших универах, куда берут только генетически качестчвенных дочек рогозина либо тех кто осилил экзамен на 70%

Аноним 19/10/21 Втр 22:05:53 547720261
>>547558
чисто жировую пищу трудно переварить. особенно если это не эмульсия молока, а кусок сала в шоколаде.

считается предварительно , что чисто беллкоковая диета тоже трудная. больше 100грамм белка якобы иногда повреждает почки, создавая нагрузку по линии выведения азотистых отходов белка в форме мочи.

углеводы же если добавить их в белково-жировую диету, снизят нагрузку на пучащий живот переваривающий пол килограмма сала и на почки выссывающие три килограмма сожранного мяса

(теоретек. фанат научпоп популяризатора нутриологии - lola fae)
Аноним 19/10/21 Втр 22:10:58 547722262
>>547558
добавка.

углеводы важны как питание мозга.наряду с кетонами. в условиях наличия гематоэнцефалического барьера.

углеводы распространены в природе в смесях с белками и жирами. поэтому организму трудно от них отказаться. все равно источник пищи белково-углеводно-жировая органика + у тебя уже есть все системы обмена углеводов. Сейчас на человека не действует антиуглеводный естественный отбор. в московских пятерочках углеводные арбузы никто не травит почти. за потребление сахара не штрафуют и не убивают.
Аноним 19/10/21 Втр 22:17:14 547723263
>>547576
окислителем. во фторе наверно теплее горит. вообще температура зависит от уравнения химической реакции. сколько энергии потенциальной в горючем и окислителе. сколько энергии тепловой и потенциальной в продукте горения
Аноним 19/10/21 Втр 22:42:45 547724264
>>547710
>Обществу выгодно поддерживать семьи.
Долбаёб пиздобольный, какая выгода если они убыточны и требуют дотаций, а без дотаций и льгот не могут существовать.
Аноним 19/10/21 Втр 23:01:34 547725265
>>547724
Общество складывается из семей, дегенерат. Общество, которое не поддерживает само себя, просто не выживет. К тому же не все семьи нуждаются в социальной поддержке. Чем выше уровень жизни в стране, тем меньше таких семей. Вывод: нужно повышать уровень жизни в стране. А чтобы повысить уровень жизни в стране, нужна нормальная власть, которая проведет нормальные экономические и социальные реформы, а не кучка жуликов, которые занимаются лишь попилом бюджета. Но это уже тематика совсем другой доски, нас тут за такое моча сейчас забанит нахуй.
Аноним 19/10/21 Втр 23:07:46 547726266
11.PNG 85Кб, 1477x1853
1477x1853
22.png 67Кб, 1485x1907
1485x1907
>>547539
Бамп вопросу.

Семь раз статически неопределимая балка. Первая пикча обычная софтина без конечных элементов (расчет мгновенный без итераций и поиска residual), вторая с конечными элементами (разбивка на пару секунд дольше, процесс схождения). Какой чисельный метод расчета используется в обычных софтинах без мкэ?
Аноним 19/10/21 Втр 23:36:53 547727267
>>547725
Ааа, так ты хуесосный пиздобольный коммунист, хуй тебе защеку, пидорок. Ещё и русский ванька-хуесос наверняка.
Аноним 19/10/21 Втр 23:41:19 547728268
tmpEULNI.png 203Кб, 409x409
409x409
>>547726
Слыш, чепуш, писавший охуенную программу для НАУЧНОЙ РАБОТЫ, а как тебе компьютер построит ту криволинейную хуёвину без итераций?
Аноним 20/10/21 Срд 01:02:11 547732269
Можно ли воссоздать отсуствие гравитации у нас на Земле чтобы полетать в невесомости?
Аноним 20/10/21 Срд 02:35:47 547733270
>>547710
> они не могут даже потенциально ни рожать
Какая разница между потенциально могут или не могут, если результат один и тот же?
> ни нормально воспитывать детей
Нормально это как? И почему не могут?

>>547732
Гравитацию экранировать никто не умеет, но есть специальный самолёт, который взлетает высоко, а потом падает вниз, и пока он падает, в нём невесомость.

Ещё можно в скафандре залезть в бассейн. У человека плотность тела примерно равна плотности воды, поэтому тоже сорт оф невесомость получается, правда в воде можно плавать, отталкиваясь от неё, в воздухе такое бы получалось куда хуже, а в вакууме не получалось бы совсем.
Аноним 20/10/21 Срд 03:08:52 547735271
>>547717
>ты делаешь акцент на генетическое качество

нет, вопрос качества не стоит

я хочу

>наиболее отличной генетикой

самую генетически дальнюю от меня тянучку
Аноним 20/10/21 Срд 04:18:34 547740272
>>547735
ой. отличной то есть не прекрасной, а различной. тогда тебе в помощь филогенетические деревья.

если судить по y-хромосоме. тебе нужна бушменка из Ботсваны-намибии-северной юар, зимбабве. либо какая то мутная народность из судана.

если судить по митохондриальной дна, наиболее удаленной от твери, тебе нужна бушменка, либо пигмейка.

суммарно по двум картам генетики человеческих популяций - тебе нужна бушменка. конечно шимпанзе и собаки еще более генетически отличаются. но с ними невозможна беременность. бушменка -баланс возможности родов и генетической отдаленности для тверчанки
Аноним 20/10/21 Срд 09:34:39 547744273
>>547120 (OP)
Можно ли каким-то образом узнать где была Планета в определенное время в прошлом?
Аноним 20/10/21 Срд 09:47:21 547745274
Аноним 20/10/21 Срд 10:52:07 547746275
>>547744
Если это единичная планта посреди нихуя посреди нихуя, то можно, там же где и была. Иначе нет, гугли "задача трёх тел"
Аноним 20/10/21 Срд 12:28:56 547749276
>>547667
Происходит одно расщепление, число вариантов - по числу возможных исходов. Когда детектор возле щели и когда детектор на экране - 2 разных эксперимента с разными возможными исходами.
>>547682
Потому и не советую загоняться.
Аноним 20/10/21 Срд 12:36:26 547750277
>>547724
> если они убыточны
Как ты это посчитал, расскажи?

У нас и так большинство семей однополые - мама с бабушкой. Они выращивают дегротов, двачеров и анимешников. В полноценных семьях вырастают альфа-Ерохины, которые больше работают, более успешны и платят больше налогов.

Это ты посчитал?
Аноним 20/10/21 Срд 13:19:09 547751278
>>547727
Какой коммунист? Ты там говна въебал что ли? Таблетки прими, пиздоглазый унтерменш.
>>547733
>Какая разница между потенциально могут
Не могут иметь детей, даже если бы захотели. Мужчины не могут рожать.
>Нормально это как? И почему не могут?
Нормально - это полноценное воспитание в полной семье, т.е. когда есть отец и мать. Для психологического развития ребенка важно, чтобы у него была полная семья. Очевидно, два пидорка или двое лесбух не могут быть нормальными родителями. Мужик не может быть матерью, а женщина не может быть отцом. Никакого нормального воспитания два мужика или две бабы ребенку дать не могут, это даже звучит абсурдно - "семья" из двух "пап" или двух "мам". Пиздец же полный.
Аноним 20/10/21 Срд 13:30:12 547752279
>>547751
>Для психологического развития ребенка важно, чтобы у него была полная семья. Очевидно, два пи
дорка или двое лесбух не могут быть нормальными родителями.
Можно пруфы? Потому что пока что, есть только обратные пруфы. Дети из однополых семей более здоровые как физически так и психологически, ибо средняя гетеро-семья это семья алкашей или залетные, а вот однополые семьи все прошли через сито отбора, поэтому детям там в среднем лучше.
Аноним 20/10/21 Срд 14:44:06 547753280
>>547752
Пидорок, спок. Иди пососи хуец своим папам.
Аноним 20/10/21 Срд 14:55:39 547754281
>>547752
>>547752
>Потому что пока что, есть только обратные пруфы. Дети из однополых семей более здоровые как физически так и психологически
Покаджи эти пруфы.

>>547752
> средняя гетеро-семья это семья алкашей или залетные,
И вот на это пруфы покажи.
Аноним 20/10/21 Срд 15:00:12 547755282
image.png 74Кб, 848x408
848x408
>>547754
>> средняя гетеро-семья это семья алкашей или залетные,
>И вот на это пруфы покажи.
Забавно, что наблюдая, как самые многодетные семьи это семьи алкашей и цыган, ты всерьез удивляешься, что средний ребенок рождается в пиздеце?
Аноним 20/10/21 Срд 16:05:18 547757283
>>547750
>Как ты это посчитал, расскажи?
Тупая долбаёбка, им платят льготы и пособия на выблядков блять, для выблядков бесплатные школы, садики, медицина, потому что их дегенераты-родители не могут их содержать сами.
Это мусорные гены, для генофонда было бы лучше если бы они все передохли.
Аноним 20/10/21 Срд 16:06:46 547758284
>>547744
Конечно можно, это довольно точно рассчитывают по теперяшнему её движению и движению соседних планет.
Аноним 20/10/21 Срд 16:20:46 547759285
>>547758
Это исключительно частный пример и по временному диапазону, и по объекту расчётов.
Аноним 20/10/21 Срд 16:34:36 547760286
>>547740
>бушменка -баланс возможности родов и генетической отдаленности для тверчанки

Слишком всратые, какие еще варианты?
Аноним 20/10/21 Срд 17:27:32 547762287
>>547751
с точки зрения науки
самые адекватные дети вырастают в семье без матери. в них дети меньше курят пьют например.
-чуть хуже дети из полной семьи.
-значительно хуже дети из семьи без отца.
-самые некачественные дети получаются из детдома с живыми родителями. (с мертвыми лучше)

перпвое и последнее меня удивило когда увидел стату
Аноним 20/10/21 Срд 17:29:58 547763288
>>547760
самая отдаленная от твери живая популяция людей - бушмены.

второе место пигмеи. тут уже есть новые плюсы, например тян пигмейка с низким ростом. многим нравится когда тян миниатюрная. наверняка и пипка узенькая.
Аноним 20/10/21 Срд 17:57:05 547766289
>>547760
Тебе генетическое разнообразие соблюдать (рожать тупых всратых недомерков) или выёбываться?
Аноним 20/10/21 Срд 18:07:33 547767290
>>547751
Долбаёбка коммунистическая, хули ты порвалась хуесоска?
Аноним 20/10/21 Срд 18:25:21 547769291
Аноним 20/10/21 Срд 18:36:26 547770292
>>547762
Мелкобуквенная нечисть, иди на хуй.
>>547767
Унтерменш, спок, я антикоммунист. Я хуй знает, что ты там употребляешь, отчего у тебя потом такие галлюцинации. Наверное бояру, как и все остальные скотоублюдки-пидорашки.
Аноним 20/10/21 Срд 23:04:24 547777293
>>547757
А ты ещё и образование с медициной платными сделать не хочешь? Или прямо сразу ценз на жизнь? Ну а чё: заработал за год меньше 1 млн - эвтаназия. Пусть только успешные выживают, а?
Аноним 20/10/21 Срд 23:08:28 547778294
Аноним 20/10/21 Срд 23:45:16 547779295
Аноним 21/10/21 Чтв 02:55:21 547788296
>>547777
А так и будет, только в масштабах государств, с полным вырезанием слабого населения, причём не только слабого в медицинском понимании, а и слабого духом и аналитически-которые сами не вырезали своих слабых соплеменников а наоборот им помогали. Например какими-нибудь китайцами каких-нибудь русских, европейцев и т.д.
От этого не уйти, та кажущаяся стабильность слабости и куколдизма лживо называемой "человечностью и заботой, равноправием" возможно только крайне недолго в условиях временной относительной изалированности территории.
Аноним 21/10/21 Чтв 02:56:18 547789297
>>547770
Ты просто крайне тупой, а не "антикоммунист".
Аноним 21/10/21 Чтв 02:56:39 547790298
Аноним 21/10/21 Чтв 10:02:13 547799299
>>547789
Спидорашка, спок.
Аноним 21/10/21 Чтв 10:18:47 547800300
Верно ли, что климат был бы куда благоприятней, не будь Антрактиды на полюсе, тогда были бы большие вертикальные течения, улучшающие тепло/влагообмен, тогда б почти не было арктических и южных пустынь? И в динозавровы времена так и было?
Аноним 21/10/21 Чтв 10:23:37 547801301
>>547800
Нахуй иди, говно.
Аноним 21/10/21 Чтв 10:24:58 547802302
Я тут к вам с платиной одного фандома. Что такое огонь с т.з. агрегатных состояний (некоторые говорят, что плазма) или его вообще так описать нельзя и это что-то другое?
Аноним 21/10/21 Чтв 10:44:16 547803303
>>547801
бля, обидно. образованный двощер всё так же остаётся двощером с хроническим /b головного мозга. просто интересно, ты действительно получаешь удовольствие от унижения людей?
Аноним 21/10/21 Чтв 10:49:42 547804304
>>547801
Главное, ведь создали тред тупых вопросов и сами ругают за тупые вопросы. Диалектика уровня /b. Ну радуйся, ты своего добился, обидел.
Аноним 21/10/21 Чтв 11:20:39 547807305
>>547803
>>547804
>имплуин тот дегенерат хоть что-то понимает
Касательно твоего вопроса не знаю, но подозреваю, что наоборот, течения проходили бы через самый полюс и ещё сильнее охлаждались.
Хз, поищи, как оно происходит на северном полюсе, там-то материка нет. С другой стороны, там всё окружено материками
Аноним 21/10/21 Чтв 18:15:11 547829306
>>547802
Он плазма, т.к. ведёт себя как плазма, под действием магнитного и электрических полей

Другой вопрос что разве огню достаточно температуры чтобы оторвать электроны у ядер?
Ну произошло окисление, ну соединился кислород с углеродом и водородом, ну естественно нагрелся при этом, но ведь не каждое окисление и соединение атомов ведёт к отрыву электронов и образованию плазмы.
А рассматривать его ещё можно просто как нагретый излучающий газ, образованный в результате окисления углерода и водорода.
Аноним 21/10/21 Чтв 21:29:02 547844307
>>547802
Зависит от того что горит и в какой среде.
Чтобы пламенем быть плазмой, нужно чтоб в нем вещество было достаточно ионизировано. Однако химические реакции в процессе горения углеводородов или прочей органики (горение неорганики отдельный разговор) протекают за счет образования радикалов и их изменения без ионизации вещества. Конечно некоторая часть радикалов способна давать ионы, но их слишком мало по отношению к общему количеству. Пламя в этом случае можно считать как газ с огромным количеством химических фаз, а процесс горение переход к равновесию фаз.
Однако есть способы превратить пламя в плазму путем подбора подходящих условий. Например использовать специальные окислители и снизить давление среды. Окислители обеспечивают рождение ионов, а низкое давление и разряженность предотвращают рекомбинацию ионов.
Аноним 21/10/21 Чтв 21:42:38 547845308
Допустим я обнаружил в песчаном карьере захоронение людей, скелеты просто закопаны в песок полностью сохранились. У одного почему-то отпилена верхушка черепа. Мне куда-то стоить трезвонить, археологов это заинтересует?
Аноним 21/10/21 Чтв 21:49:11 547847309
16146836230970.png 3065Кб, 1600x1200
1600x1200
>>547744
нахождение планет известно с точность 4 метра. чем дальше в прошлое, тем больше эта погрешность растет
Аноним 21/10/21 Чтв 21:52:31 547848310
>>547845
В полицию позвони лучше.
Аноним 21/10/21 Чтв 21:54:59 547849311
png.png 392Кб, 606x788
606x788
Аноним 21/10/21 Чтв 23:23:24 547855312
16307624276510.png 523Кб, 800x427
800x427
>>547848
>В полицию позвони лучше.
Как правильно оценить математическое ожидание того, что полиция начнёт прорабатывать версию с устроением означенного геноцида гражданином, сообщившим о точных координатах и особенностях (отпил черепной коробки) данного захоронения в качестве генеральной версии? Как загодя корректно определить предельное и средневзвешенное потенциально негативное воздействие сотрудниками полиции на обратившегося?
Аноним 21/10/21 Чтв 23:32:48 547857313
>>547855
Когда почуешь в жопе швабру сразу во всем признаешься и подпишешь все бумаги
Аноним 22/10/21 Птн 08:58:02 547878314
not a.jpg 2Кб, 225x225
225x225
Я тут как-то уже спрашивал, мгновенно налетели дебичи с опровержениями посылки, а не с ответами на упражнение для ума. Но я спрошу ещё, почему нет.
В западной академической среде (психология, медицина, "гуманитарные науки"), а также среди большинства образованных людей принято различать пол и гендер, и вообще разделять то, что называют "гендерной теорией".
Аргументы против все знают. Какие аргументы "за"?
Это тред тупых вопросов, но не тупых ответов, если что. Если ты слишком туп чтобы сконструировать аргумент за кажущуюся слабой и не разделяемую тобой позицию - ничего не пиши."
Аноним 22/10/21 Птн 09:11:56 547879315
>>547878
>Какие аргументы "за"?
Если ты против, то ты хуже Гитлера
Аноним 22/10/21 Птн 10:06:09 547880316
>>547878
> Какие аргументы "за"?
пидоры не плодятся
Аноним 22/10/21 Птн 10:20:56 547882317
>>547788
Такая херь кончилась ещё при шумерах и ассирийцах. Ты с глиняной таблички капчуешь?
Аноним 22/10/21 Птн 10:23:34 547883318
>>547829
При окислении электрон отрывается от одного атома и переходит к другому. По определению окисления, лол.
Аноним 22/10/21 Птн 11:13:26 547886319
>>547878
Ёбаный лахтобот, ты заебал уже высирать одну и ту же хуйню из треда в тред. Репорт лахтодырому пидарасу.
Аноним 22/10/21 Птн 11:15:50 547887320
>>547878
Чтобы общество не навязывало тебе роль по наличию/отсутствию у тебя пинуса. Роль ведь подразумевает определенные ожидания от поведения индивида, и если он не соответствует ожиданиям - то общество его порицает. Таким образом раньше ты либо сжимаешь булки и соответствуешь, и если сила воли сжимать булки заканчивается, то ты начинаешь бухать, нюхать, присаживаться на антидепрессанты и тому подобное, либо ты забиваешь хуй и становишься порицаемым аутсайдером и на фоне этого начинаешь прибухивать ну ты понел. И вот эта новая левацкая идеология предостовляет опцию номер 3 - занять какое-то место на спектре гендерного многообразия, которое соответствует тебе, чувствовать себя комфортно и не быть порицаемым.
Звучит хорошо, и для кого-то наверняка будет отличным вариантом, недостаток в том что шизы и вниманиебляди моментально воспользовались ситуацией и заполонили собой медиапространство и производят так сказать черный пиар всей этой теме.
Аноним 22/10/21 Птн 12:38:57 547895321
16344206527240.jpg 122Кб, 720x720
720x720
>>547887
Разъяснил как боженька.
Аноним 22/10/21 Птн 13:52:28 547905322
>>547887
Это очень интересно, но я спросил про аргументы за валидность теории.
А ты и шизики выше высказали свои охуенно важные мета-комментарии.
Аноним 22/10/21 Птн 14:16:50 547906323
>>547905
Если хочешь говорить предметно, то называй положения теории, в пользу которых ты хочешь услышать аргументы.
Ну и в такой ситуации аргументами могут быть результаты исследований, выраженные некой статистикой. Может кто-то в треде и подбросит, мне самому интересно.
Аноним 22/10/21 Птн 14:45:13 547908324
>>547906
ИТТ уже четверо порвали себе жопы, опровергая неизвестно что, ясно.
Все прекрасно понимают, что это. Если не понимают, то зачем мне их аргументы?
Ну и научные теории прекрасно существуюти и без статистики.
Аноним 22/10/21 Птн 15:17:08 547911325
>>547908
>Ну и научные теории прекрасно существуюти и без статистики.
Для того, чтобы теория существовала достаточно опровержимости.
А вот для доказательства - нужен эксперимент. Когда дело касается людей и социума - в основном в ход идет именно статистика.
Ну и для примера возьмем бак, проводят эксперимент, получают статистику, и если отклонения не меньше чем сколько-то там, делают выводы о доказательстве или опровержении.
Аноним 22/10/21 Птн 16:54:20 547916326
1630077715630.jpg 40Кб, 680x615
680x615
Пагни, поясните, правильно ли я понял текущую мету?

Если даже относительно большие объекты можно заставить интерферировать просто лишив их взаимодействия с окружающим миром и охладив - значит именно постоянное взаимодействие нас локализует. А именно - мы локализованы до тех пор, пока мы в состоянии излучать в ИК, или еще каким-то образом взаимодействовать с окружающим миром - после этого мы становимся полноценным котом Шредингера. При этом мы все еще будем находиться в пространстве времени и на нашу функцию все еще будут влиять его искривления.

Тогда помогите понять нащупанные на данный момент границы квантовой запутанности, и если они динамичны - что именно на них влияет, что именно выводит объект из этого состояния, локализуя его? Как это согласуется с принципом наименьшей энергии, например? Ведь фактически именно состояние суперпозиции является состоянием покоя, однако если локализовать атом нонстоп - это влияет на время его распада, например - то есть ваш покой не покой, задроченный проверками локализованный атом живет дольше, чем атом интерферирующий - значит в интерферирующем атоме что-то происходит даже во время нахождения в суперпозиции. Тогда какого хуя он в этом состоянии, раз в нем что-то происходит?
Аноним 22/10/21 Птн 17:32:47 547917327
А вот если критерием вида является то что его представитель не может давать плодовитое потомство с представителем другого вида, то как определяют виды у прокариот размножающихся неполовым путём?
Аноним 22/10/21 Птн 17:52:18 547918328
>>547911
Реальная наука это не впопер, и рамочные теории не нуждаются в статистике, они - способ её интерпретации.
Жду аргументов, кароч.
Аноним 22/10/21 Птн 18:21:27 547919329
>>547917
Это один из критериев. Если он не применим то его естественно не применяют.
Аноним 22/10/21 Птн 18:23:56 547920330
>>547919
А как их тогда различают? Чисто умозрительно чтоли?
Аноним 22/10/21 Птн 18:28:31 547921331
>>547920
По биохимии например. Что жрут. Чем срут.
Аноним 22/10/21 Птн 18:56:57 547922332
>>547918
>Реальная наука это не впопер
Стоило начинать с того, что ты альтернативщик, а не с геев. Тогда бы анон не тратил на тебя время.
Аноним 22/10/21 Птн 19:17:14 547923333
>>547878
Разрушение всех идентичностей задумывалось и планомерно воплощается как дальнейшее продвижение индивидуализма для окончательного развенчания коллективистких идеологий - таких, например как коммунизм, или нацизм.
Плюсом является то, что такое общество терпимо к сексуальным девиациям.
Минусом является то, что такое общество терпимо к сексуальным девиациям.
Аноним 22/10/21 Птн 19:41:50 547924334
>>547878
>Аргументы против все знают.
Нет, расскажи. Только очко смажь сначала, и проидентифицируй его гендерно, а то оно у обоих биологических полов одинаковое.
Аноним 22/10/21 Птн 19:59:36 547925335
>>547922
Геев?
>>547923
Ты тупой лол.
>>547924
Вс самые убедительные аргументы тут уже озвучены. Захватывающее чтение, да?
Аноним 22/10/21 Птн 20:07:57 547926336
>>547925
>Вс самые убедительные аргументы тут уже озвучены.
Перечисли.
>Захватывающее чтение, да?
То, как зарплатный вертится на сковородке - да, безусловно.
Аноним 22/10/21 Птн 20:59:51 547927337
>>547926
>То, как зарплатный вертится на сковородке
Ты намекаешь, что "докажите что левацкая теория верна, а не докажите - обосрались!" - зарплатный додик?
Аноним 22/10/21 Птн 21:19:10 547928338
>>547925
>Ты тупой лол.
Или перестаешь пердеть в лужу со своими убогими оценочными суждениями и даешь конкретику, или идешь на хуй, тебя и так со всех сторон обоссали.
Аноним 22/10/21 Птн 21:55:37 547929339
>>547928
Ты очень тупой.
>>547927
Привести хорошие аргументы в пользу чужой позиции не значит её доказать или принять. Это упражнение, причём очень простое, для школьников. Пока что тут уровень детсада.
Аноним 22/10/21 Птн 22:50:03 547930340
>>547929
>Ты очень тупой.
Бремя тупых - наблюдать тебя, не способного не то, что на конструктив, даже на осознание плоскости в которой пролегает вопрос.
Аноним 23/10/21 Суб 01:48:44 547931341
Аноны, из чего следует то что тело вращающееся вокруг планеты на минимальной высоты орбите( а это значит когда у него максимальная орбитальная скорость), не может улететь в бесконечно дальний космос от этого тела?
Т.е. скорость максимальная, энергия кинетическая как скорость в квадрате.
А гравитационное притяжения ослабевает обратноквадратично.
Т.е. понятно что на подъём орбиты должна тратится энергия котлрая будет уходить в потенциальную, но ведь на бОльшей высоте и притяжение слабее и орбитальная скорость нужна меньше
Аноним 23/10/21 Суб 01:54:48 547932342
>>547931
>гравитационное притяжения ослабевает обратноквадратично
Ты правда сравниваешь энергию кинетическую и силу, а не энергию, и делаешь какой-то вывод?
Аноним 23/10/21 Суб 01:57:49 547933343
Хм, а ведь чёрные дыры не переводят массу падающего в неё тело в энергию, масса упавшая остаётся та же, а выделяется в виде излучения потенциальная энергия
Получается потенциальная энергия любых тел во вселенной это не mgh, а энергия до перехода за горизонт событий.
И получается что-то когда-то развело все эти тела дав им такую потенциальную энергию
А то что сейчас творит тёмная энергия это хуйня, по сравнению с тем что было

И вот что ещё странно, вроде как потенциальная энергия есть, но в случае падения на чёрную дыру она выделится, только если до столкновения с горизонтом оно упадёт не прямо, а теревшись в аккреционном диске обо что-то
В противном случае... Вот интересно, вроде как не может вникуда она деться
Получается если в чд падает тело массой М, со скоростью, которая даст ему массу эквивалентно энергии... получается упавшее в ЧД тело массой М увеличивает массу ЧД на 2М?
Если не тёрлось о аккреционный диск а прямо упало разогнавшись максимально возможно.
Аноним 23/10/21 Суб 01:59:40 547934344
>>547932
Энергия потеницальная появляется из-за силы притяжения
Уменьшение прироста потенциальной энергии при увеличении отдаления от гравитирующего тело происходит также как ослабляется сила притяжения.
Аноним 23/10/21 Суб 02:04:29 547935345
>>547934
Сила это производная от энергии по координатам. Силы всегда имеют на одну степень ниже зависимости.
Аноним 23/10/21 Суб 02:07:30 547936346
>>547935
Ну перевернул ты, и что изменилось?
Аноним 23/10/21 Суб 02:18:12 547937347
>>547936
1/r или 1/r^2 по твоему никакой разницы?
А вообще, как твое тело собирается улетать имея полную энергию ниже нуля?
Аноним 23/10/21 Суб 02:32:29 547939348
>>547937
Ну смотри, высота около 0, т.е. потенциальная энергия 0
При этом кинетическая максимальна
На бесконечном же удалении потенциальная энергия максимальна, а орбитальная скорость 0
Как понимаю кинетическая энергия на околонулевой высоте равна максимально возможной потенциальной

Т.е. мне непонятно откуда берётся интервал начальных скоростей спутников, для земли вроде от 8 до 15 км/с
Типа на минимальной высоте 8, на максимальной начальная 15 уходящая в 0 при подъёме
Почему нельзя кружась по орбите на минимальной высоте поднять высоту орбиты не затрачивая энергию, а просто затратив скорость, ведь на бОльшей высоте орбитальная скорость нужна меньше
Типа просто взял обменял скорость на высоту орбиты
Аноним 23/10/21 Суб 04:36:09 547942349
>>547939
>Ну смотри, высота около 0, т.е. потенциальная энергия 0
Ноль на бесконечном расстоянии. Пересчитывай
Аноним 23/10/21 Суб 09:17:10 547945350
>>547762
Можешь показать какие-то исследования или статистику? Интересно читнуть.
Аноним 23/10/21 Суб 11:30:55 547948351
>>547762
>-самые некачественные дети получаются из детдома с живыми родителями. (с мертвыми лучше)
Собсно, в детдом с живыми родителями можно попасть если они оба наркоманы/рецидивисты. Сюрприз-сюрприз низкий интеллект и импульсивное поведение хорошо наследуются.
>-значительно хуже дети из семьи без отца.
Твоя статистика скорее всего из США, и там чуть ли не треть таких детей - негры (т.к. чуть ли не у 90% всех негритянских детей нет отцов). А у негров свои проблемы.
>самые адекватные дети вырастают в семье без матери
Учитывая, что таких семей практически нет ни в одной стране это просто статистический шум.
Таким образом приходим к тому, что "статистика" нихуя не значит, и социальным наукам нужен рамочный анализ. %%Как гендерная теория.
Аноним 23/10/21 Суб 12:59:35 547950352
image.png 1762Кб, 800x800
800x800
Какой шанс обнаружить новых представителей мегафауны, которые будут сильно отличаться от известных?
Аноним 23/10/21 Суб 13:35:09 547951353
>>547120 (OP)
Есть 2 теории происхождения нефти. 1 - биогенная. Анаэробная атмосфера содержала СО2 которые автотрофы превратили в органику. При этом часть получившегося кислорода пошло на окисление железа и прочих пород. Получается что по стехиометрии мы в принципе не можем сжечь всю нефть/газ/уголь, потому что не хватит кислорода?
А если альтернативные теории рассматривать, например карбидную, то получается ископаемого топлива еще больше?
Аноним 23/10/21 Суб 14:38:01 547953354
Аноним 23/10/21 Суб 14:52:16 547954355
>>547953
Тимати с гастролями приезжал
Аноним 23/10/21 Суб 19:27:07 547959356
>>547942
Ну ты опять просто перевернул нихуя не изменив.
Кстати, какого хуя считают что потенциальная энергия в случае тела которое притягивает гравитация убывает при увеличении высоты?
Аноним 23/10/21 Суб 19:35:07 547960357
>>547959
Не убывает. Ты просто знак забыл
Аноним 23/10/21 Суб 19:39:56 547961358
>>547960
Чем отличается сказать растёт со знаком -, или сказать убывает со знаком +?
Аноним 23/10/21 Суб 19:47:53 547962359
>>547961
Ты там траллишь меня блять? Ты сам то понял что спросих хуйню? У тебя независимо от точки нуля потенциальной энергии, она растет при удалении.
Аноним 23/10/21 Суб 19:54:20 547965360
>>547942
>Ну смотри, высота около 0, т.е. потенциальная энергия 0
>Ноль на бесконечном расстоянии. Пересчитывай
Ну а это что за пук был тогда?
Аноним 23/10/21 Суб 19:54:48 547966361
Аноним 23/10/21 Суб 19:58:11 547967362
>>547965
Классически принято, брать ноль энергии на бесконечной расстоянии, а не на нулевой высоте (так удобнее). Если же ты хочешь ноль на нулевой высоте (ну такой ты извращенец), то тогда у тебя на бесконечном расстоянии будет энергия положительная (но не бесконечная).
Аноним 23/10/21 Суб 20:30:33 547969363
>>547967
Короче обычная тофтология и ничего по теме

>высота около 0, т.е. потенциальная энергия 0
>При этом кинетическая максимальна
>На бесконечном же удалении потенциальная энергия максимальна, а орбитальная скорость 0
>Как понимаю кинетическая энергия на околонулевой высоте равна максимально возможной потенциальной

>Т.е. мне непонятно откуда берётся интервал начальных скоростей спутников, для земли вроде от 8 до 15 км/с
>Типа на минимальной высоте 8, на максимальной начальная 15 уходящая в 0 при подъёме
>Почему нельзя кружась по орбите на минимальной высоте поднять высоту орбиты не затрачивая энергию, а просто затратив скорость, ведь на бОльшей высоте орбитальная скорость нужна меньше
>Типа просто взял обменял скорость на высоту орбиты
Аноним 23/10/21 Суб 20:32:33 547970364
>>547969
Ничего по теме, потому что я верю в людей, и верю, что ты сам сможешь понять, сложив 2+2 откуда берется граница для скоростей.
Аноним 23/10/21 Суб 20:34:42 547971365
>>547970
Ну а нахер тут твой высер про веру?
Аноним 23/10/21 Суб 21:32:22 547972366
>>547971
Если потенциальная энергия у тела на орбите условных -20 единиц, а кинетическая энергия 10 единиц.
Вопрос 1, какая полная энергия.
Вопрос 2, какая граница для потенциальной и для кинетической энергии?
Аноним 23/10/21 Суб 22:44:56 547973367
Аноним 23/10/21 Суб 22:54:28 547974368
Аноним 23/10/21 Суб 23:05:11 547975369
>>547969
Ты можешь брать нуль потенциальной энергии где хочешь. Обычно берут на бесконечности, потому что так удобнее. Но если ты особенный, можешь брать не там.
Аноним 23/10/21 Суб 23:09:16 547976370
Аноним 23/10/21 Суб 23:10:58 547977371
>>547975
Так какой в этом смысл, если тело начнёт падать, и чем выше начнёт, тем больше перейдёт в кинетическую энергию. Значит чем выше тем больше потенциальной энергии, в этом же и смысл потенциальной энергии.
Аноним 23/10/21 Суб 23:24:41 547978372
>>547976
Даже моя мелкая сестра знает, что 10-20 это -10.
Аноним 24/10/21 Вск 00:14:31 547980373
>>547978
Вот только ты такой же тупой как твоя мелкая сестра и не знаешь что энергия не так складывается.
Аноним 24/10/21 Вск 00:19:21 547981374
>>547980
Ясно, это тред тупых вопросов, троллить иди в Б
Аноним 24/10/21 Вск 00:25:56 547982375
>>547981
Ага, пиздобол будет мне доказывать что у тела на орбите энергия не равна сумме по модулю потенциальной и кинетической.
Конечно ей воспользоваться такой не удастся напрямую, но вот если добавить ещё кинетической энергии и вывести его на высоту когда потратится достаточное количество скорости орбитальной, то он пизданётся с энергией как СУММА потенциальной и кинетической энергий, по модулю.
Аноним 24/10/21 Вск 01:09:24 547983376
>>547982
Сходи в учебник или на википедию. Я не пытаюсь подъебать, мне просто стыдно от тебя.
Аноним 24/10/21 Вск 01:29:49 547984377
>>547983
Ты б хотяб мои слова опровергнуть попытался, а не так позорился безпруфно.
Аноним 24/10/21 Вск 01:30:33 547985378
>>547984
Это ниже критики
Аноним 24/10/21 Вск 01:41:23 547986379
>>547985
Просто ты в критику не можешь, умишка не хватает.
Аноним 24/10/21 Вск 02:59:50 547987380
Поясните про эру докислородной атмосферы.
Тогда что, тупо почти на 100% атмосфера из СО2 и азота состояла? И метана вроде ещё дофига было, но он не окислялся.
Типа весь СО2 сейчас в связанном виде? Но ведь кислорода пиздец как много, а углерода как понимаю мало, да и должен какой-то баланс соблюдаться, не может быть в связанном виде углерода сильно больше чем его в атмосфере.
Аноним 24/10/21 Вск 03:57:27 547988381
>>547987
Солнце светило, и эту энергию потратили микроорганизмы на создание кислорода.
Аноним 24/10/21 Вск 04:30:28 547989382
>>547988
Такого вопроса нет в посте.
Аноним 24/10/21 Вск 05:31:20 547991383
>>547987
>Тогда что, тупо почти на 100% атмосфера из СО2 и азота состояла?
Да. Азот, углекислота, синильная кислота, пары воды.
>Типа весь СО2 сейчас в связанном виде? Но ведь кислорода пиздец как много, а углерода как понимаю мало
Кислород в атмосфере не из углекислоты растения делают, а из воды.
Аноним 24/10/21 Вск 10:33:10 547997384
Аноним 24/10/21 Вск 10:35:18 547998385
>>547977
Считать удобнее.

Я говорю, можешь брать начало где хочешь. Главное в начале всем объяснять, где ты его взял, и не менять в процессе.
Аноним 24/10/21 Вск 13:00:12 548000386
>>547950
Ты можешь найти какую-то нёх глубоко в океане, но и то далеко не факт.
Аноним 24/10/21 Вск 20:17:17 548009387
5a8912a3f8ec9a0[...].png 23Кб, 694x566
694x566
7a66761777fa82e[...].jpe 128Кб, 1180x800
1180x800
Даже не знаю в каком из тредов тупых вопросов спросить. Удостою этой чести наукач.
Задача создать циркуляцию воды в пруду.
Возможно ли с помощью каких-то хитрых коленвалов(?), использовать подвижный ветряк/лопасти (всмысле, что бы он по ветру крутился) и передавать это на винт под водой.
Или это реализуемо только с ветряками типа пик 2?
Аноним 24/10/21 Вск 20:27:32 548010388
>>548009
>Задача создать циркуляцию воды в пруду.
Чем тебя насос не устраивает?
Аноним 24/10/21 Вск 20:31:44 548012389
>>548010
Зачем мне лишний раз обогащать капитаглиста, если я могу из говна и палок сделать тоже самое, используя нашу с вами силу природы?
Аноним 24/10/21 Вск 20:33:52 548013390
>>548012
Метал тоже будешь сам лить, или все таки на поклон к торговцу алюминием пойдешь?
Аноним 24/10/21 Вск 20:44:12 548014391
Аноним 24/10/21 Вск 20:46:12 548016392
>>548013
На помойке найду лист алюминия и сделаю из него лопасти.
Но и ты в сторону сильно не уходи. Одно дело один раз купить у капитаглиста лист алюминия, другое дело постоянно платить за электроэнергию.
Аноним 24/10/21 Вск 20:51:03 548017393
>>548014
Что блядь делать, мудила ты ебаная? Я и зашел сюда спросить конкретный вопрос, можно ли с движущегося ветряка передать энергию на винт. Пост на который ты пукнул к делу отношения не имеет.
Аноним 24/10/21 Вск 20:59:07 548018394
Аноним 24/10/21 Вск 21:04:18 548020395
Аноним 24/10/21 Вск 21:16:49 548021396
Аноним 24/10/21 Вск 21:17:39 548022397
>>548020
На каком этапе у тебя возникла проблема?
Аноним 24/10/21 Вск 21:32:26 548023398
>>548022
Я представляю, как шестеренками или коленвалом передать энергию с неподвижного винта передать энергию на винт под водой. Но не могу представить, как со свободно вращающегося винта передать. И даже не представляю, что мне гуглить, что бы узнать.
Аноним 24/10/21 Вск 22:49:44 548026399
>>548023
Погугли "дифференциал", "коническая зубчатая пара". Может зависить что у тебя там вместо ветряка.
Аноним 25/10/21 Пнд 00:31:04 548029400
>>547991
А из СО2 почему кислород не считаешь как тот "что делают растения"?
Аноним 25/10/21 Пнд 01:06:58 548030401
123
Аноним 25/10/21 Пнд 01:07:19 548031402
Аноним 25/10/21 Пнд 07:59:39 548032403
Где найти статистику по росту (длине тела) населения РФ (и желательно РСФСР) в перцентилях? Хотя бы новейшую по РФ.
Аноним 25/10/21 Пнд 08:36:08 548033404
unnamed.jpg 25Кб, 448x299
448x299
Аноним 25/10/21 Пнд 10:52:29 548040405
Если сильно увеличить скорость прохождения нервных импульсов, то человек станет воспринимать мир словно в ускоренной съемке?
Аноним 25/10/21 Пнд 10:53:32 548041406
>>548029
>лород не считаешь как тот "что делают ра
Ты про что?
Аноним 25/10/21 Пнд 12:32:01 548045407
У меня есть лазерная указка с минифонариком. Почему Лазер оттуда светит в виде точки, а фонарь - в виде конуса? Почему у лазера фотоны не разлетаются в стороны?
Аноним 25/10/21 Пнд 12:37:21 548046408
>>548045
Разлетаются, просто ПОМЭНШЭ.
Аноним 25/10/21 Пнд 12:51:01 548048409
>>548045
Там настоящий лазер?

>>548040
Реакция улучшится и всё.
Аноним 25/10/21 Пнд 16:10:58 548059410
сап аноны.подскажите как понять что тот или иной предмет не дается в силу того что ты мало на него времени тратить а в силу того что то ты просто не подходишь для этого предмета?
Аноним 25/10/21 Пнд 16:34:52 548060411
>>548059
Кора больших полушарий человека это по сути набор из специализированных нейросеток. Совершенно точно есть такие сферы умственного труда, которые ты не потянешь. В то же время у тебя может быть божественный талант играть на ксилофоне.
Аноним 25/10/21 Пнд 17:05:08 548062412
>>548045
Так в этом же суть лазера в формирование плоской когерентной электромагнитной волны, а обычной фонарик создает сферические волны.
Когерентность и плоскость волн лазера создается за счет квантового вынужденного излучения. Суть которого в способности возбужденной среды "копировать" фотоны, которые ее "разряжают". При копирование же строго задаются начальные данные фотонов, что выражается в плоскостности волны.
При излучение фонариков проходит через спонтанное изучение и начальные параметры строго не определены и гуляют вокруг среднего, что в конечном случае выражается в сферичности волны.
Аноним 25/10/21 Пнд 18:21:19 548063413
>>548062
Дядь объясни по простому, как будто мне 7 лет.
Аноним 25/10/21 Пнд 19:45:34 548066414
16327571319280.jpg 92Кб, 640x622
640x622
>>548023
>Но не могу представить, как со свободно вращающегося винта передать. И даже не представляю, что мне гуглить, что бы узнать.
Гугли: "мельницы средневековья бесплатно без СМС".
Аноним 25/10/21 Пнд 19:55:34 548068415
16339451451051.jpg 93Кб, 640x464
640x464
>>548060
Сергей Вячеславович, Вы нам в палках колбасы объясните!
Аноним 25/10/21 Пнд 21:19:34 548073416
>>548041
>>547991
>Кислород в атмосфере не из углекислоты растения делают, а из воды.
>>548029
>А из СО2 почему кислород не считаешь как тот "что делают растения"?
Ведь растения из окружающей среды не только h2о берут но и со2
Аноним 25/10/21 Пнд 22:52:28 548083417
>>548073
CO2 используется для производства сахаров. Кислород только из воды получается.
Аноним 26/10/21 Втр 10:29:17 548097418
>>547229
Чтобы получить большую температуру, нужно сконцентрировать излучение, излучаемое предметом большой площади на предмет меньшей площади. Однако проблема излучателя большой площади в том, что из каждой точки его излучающей поверхности излучение летит во все стороны. Я не могу это доказать математически (тупой слишком), но путём построения простых двумерных моделей обнаружил, что не могу сконцентрировать такое излучение на меньшей площади, чем площадь излучателя. Соответственно нагревать что-то сильнее чем излучатель -- не получается.
И я не один такой умник. За долго до меня такое пытались. Даже написали фантастический роман "Гиперболоид инженера Гарина", в котором инженер Гарин якобы смог сконцентрировать тепловое излучение. Однако фантастика на то и фантастика, что описывает невозможные вещи.
Если бы же такой фокус кому-то удалось бы провернуть, то этот человек изобрёл бы тепловую машину с охуительно высоким КПД, выше чем существуют сейчас.
Аноним 26/10/21 Втр 10:54:35 548098419
Как же вы заебали своими квантами и чорными дырами.
Аноним 26/10/21 Втр 15:07:46 548103420
Аноним 26/10/21 Втр 15:20:36 548104421
>>548060
так в этом и вопрос как понять у тебя не получается из за того что не тянешь или из за того что ты ленишься?
Аноним 26/10/21 Втр 16:19:06 548109422
>>548104
Ты совсем тупой и не можешь понять интересно тебе то чем ты пытаешься заниматься или нет?
Аноним 26/10/21 Втр 16:19:56 548111423
Аноним 26/10/21 Втр 16:22:06 548113424
>>548111
Значит не тянешь.
Аноним 26/10/21 Втр 16:45:37 548116425
>>548109
мне вот было бы интересно заниматься построением космических ракет и как ты думаешь могу ли я потянуть подобное?
Аноним 26/10/21 Втр 17:01:04 548117426
>>548116
Если реально интересно, и не будет раздражающих мещающих факторов, типа пидарашек вокруг, то да.

Но думаю это маняфантазия у тебя а не реальный интерес к этому. Подавляющее большинство макакенов хотят просто быть богатым, знаменитым, иметь власть, машину, яхту, шалав, пить, жрать, вниманиеблядствовать и социоблядствовать.
Аноним 26/10/21 Втр 21:26:20 548130427
Нахуя говорят что электрон и другие элементарные частицы точечные?
Просто зачем? Какой в этом смысл?
Аноним 26/10/21 Втр 21:29:12 548131428
>>548117
>Но думаю это маняфантазия у тебя а не реальный интерес к этому
а как отличить маняфантазии от реального интереса?какой процент твоего свободного времени должен на этой уходить?
> Подавляющее большинство макакенов хотят просто быть богатым, знаменитым, иметь власть, машину, яхту, шалав, пить, жрать, вниманиеблядствовать и социоблядствовать.
ну это очень хорошие мотиваторы если их грамотно использовать.
Аноним 26/10/21 Втр 21:44:30 548132429
>>548131
>а как отличить маняфантазии от реального интереса?какой процент твоего свободного времени должен на этой уходить?
Учись в самооанализ, я ж не буду копаться в говнище которое у тебя в голове.
Время не показатель, чекай свои жизненные установки, то что считаешь реально нужным и важным.
>ну это очень хорошие мотиваторы если их грамотно использовать.
Ну так речь не о том на что ты готов за мотиваторы, а чего ты хочешь.
Если ты считаешь эти вещи мотиваторами, то наверное это не то чего ты хочешь.
Хотя тут от определения зависит, может "мотивтор" в твоём определении может включать в себя и смысл жизни, а не просто то для чего "впринципе можно было бы обменять часть своего времени работая или делая что-то ещё".
Аноним 26/10/21 Втр 21:46:56 548133430
>>548063
Ну епты сидят електроны на энергетических уровнях, им пизды дают напряжением, они через короткое время обратно падают на свои уровни, пукая фотоном с энергией, равной высоте с которой упал електрон. То, что уровни энергетические дискретны и можно довольно точно закидывать електроны на нужные уровни выливается в то, что длину волны можно получать пизда как точно.

А вот фонарик светит белым светом, в котором дохуилиард волн разной длины и поэтому получается хуета. Мвоут.

мимопроходил
Аноним 26/10/21 Втр 21:53:31 548134431
>>548133
Это ничего не говорит о направленности излучения и его расхождении

А, вы про преломление в воздухе и расхождение из-за этого, дошло
Аноним 26/10/21 Втр 22:02:12 548135432
Аноним 26/10/21 Втр 22:12:27 548136433
>>548135
Найди принципиальную схему лазера.
Аноним 26/10/21 Втр 22:30:46 548138434
>>548134
Не, тут ещё дело в том, что в лазере большинство электронов получили пизды, и сидят на высоких уровнях. Когда рядом с ними пролетает фотон той же энергии, что и разность между уровнями, они могу с какой-то вероятностью упасть, при этом рождается ещё один фотон, который при этом ещё и является копией того, что рядом пролетал. В лазерах ещё есть два зеркала, из-за которых фотоны гуляют туда-сюда вдоль оси лазера, постепенно накапливаясь, вот и получается плоская когерентная волна
Аноним 26/10/21 Втр 22:32:54 548139435
>>548138
В лазере нет оптики фоккусирующей вообще?
Как добиваются того, вернее, какой физический принцип того что фотоны вылетают исключительно перпендикулярно поверхности излучающего материала?
Аноним 26/10/21 Втр 22:36:18 548140436
>>548138
Ты так говоришь, будто там чисто одна волна выходит, но вещь это не так
И про фонарик писал что там круговая волна выходит, но ведь и это не так
Фонарик излучает просто много фотонов направленных в разные стороны
И лазер тоже, не одну волну, а много похожих фотонов направленные в одну сторону.
Аноним 26/10/21 Втр 23:24:48 548141437
16293139731100.jpg 72Кб, 676x661
676x661
>>548139

Оптика есть, но основным принципом служит то, что лазере всё излучение с одинаковой длиной волны, следовательно каждый отдельный фотон имеет минимальную разницу в углах отклонения при преломлении и отражении относительно других фотонов в пучке.

нихуя не знаю о лазерах, просто мимо проходил
Аноним 26/10/21 Втр 23:30:36 548142438
>>548141
Не тянет на "основной принцип". Если там излучение в разные стороны, то похер на ощинаковость длины волн, их почти также сложно будет сфокусировать как и в фонарике.
Аноним 26/10/21 Втр 23:58:00 548143439
>>548142
В таком случае могу предложить сонаправленность излучающих атомов, что позиционируются в зависимости от направления электрического тока.

Еще можно предположить сонаправленность с помощью кристаллической решетки, но насколько я знаю есть лазеры, активная зона которых содержит газообразную среду.

Короч теперь я сам запутался, и завтра погуглю.
Аноним 27/10/21 Срд 00:10:36 548144440
>>548130
>Нахуя говорят что электрон и другие элементарные частицы точечные?
Потому что они являются точечными объектами, т.е. обладают нулевым размером.
Аноним 27/10/21 Срд 00:19:09 548145441
jCZ1gK8JWY.jpg 98Кб, 862x802
862x802
>>547120 (OP)
Предположим, что в неё добавили гипотетический ароматизатор с запахом глазури, который является абсолютно безвкусным. Будем ли вода на вкус как глазурь? И если да, то почему так будет происходить
Аноним 27/10/21 Срд 00:19:47 548146442
Аноним 27/10/21 Срд 00:50:33 548147443
>>548145
Ты её в нос зальёшь? Ты никогда не встречал веществ у которых отличается запах и вкус?
Аноним 27/10/21 Срд 00:52:14 548148444
>>548144
1) Нахуя, где это свойство вылазит?
2) Как у тебя волна может быть точечной, ты ебанулся?
Аноним 27/10/21 Срд 00:58:03 548149445
Вот типа размерная частица, ты её хуяришь, до приближаясь к границе её "формы", до тех пор пока в очередной раз для ещё большего приближения нужно будет ухуярить так, что она развалится, т.е. перстанет быть именно той частицей которую исследовали. Т.е. очевидно это её размер.
Но теперь берём кусок резины и хуярим его, ты можешь дойти до его центра "ударами", т.е. промять его прям до центра, но будет очевидным бредом сказать что у этого куска резины нулевой размер, он просто податливый и не разрушается, деформируясь при ударах, отодвигая от удара часть себя в ту сторону где удара не было.
Аноним 27/10/21 Срд 04:36:23 548184446
>>548130
Штобы точно ебашить из электронной пушки. Раньше так телевизоры и мониторы работали.
Аноним 27/10/21 Срд 04:38:32 548185447
Аноним 27/10/21 Срд 04:42:47 548186448
>>548185
А то что электрон нихера не точечный а просто деформируется.
Аноним 27/10/21 Срд 04:48:55 548188449
>>548186
Если бы было так, то в уравнениях рассеивания электронов возникали бы разные хуевины. Ты думаешь такой умный? Ученые уже дохуя раз предполагали что "а что если электрон не точечный, к чему это приведет? А... картинка в этих экспериментах изменится, хорошо, а может иначе?" И рефлексировали этот вопрос очень много раз. Целый учебник на 1500 тысяч страниц можно написать, который будет весь посвящен разным тезисам и проверками о размерах электрона. В каждом учебнике по КЭД даже отдельная глава есть на эту тему.
Аноним 27/10/21 Срд 04:50:49 548189450
>>548186
Ты там смотри аккуратней электроны деформируй. А то сломаеш.
Аноним 27/10/21 Срд 04:59:03 548192451
>>548189
А он похоже неуничтожим, только сам чд стать может от слишком сильного воздействия, или провалиться за горизонт, причём с сохранением заряда.
Странно очень кстати что заряд неуничтожим
Скорее всего это показывает что всё едино, электромагнитное поле, электронное, пространство/гравитация и всё остальное.
И теории разделяющие всё это неверны. В том смысле что будут иметь более худшие предсказательные свойства чем теория, описывающая все эти сущности как разные проявления одного целого.
Аноним 27/10/21 Срд 05:05:18 548193452
>>548188
И что там в этих главах пишется подробнее?

Как там описывается отсутствие деформации у электрона при столкновении? И других воздействиях.

Т.е. очень странным мне кажется было бы поведение, когда условный "сгусток" поля, даже если он точечный, якобы, вокруг него всё-равно потенциал взаимодействия по какому-то закону изменяющийся от условного "края" к условному "центру". Так вот, удар и "промятие" с края до центра с одной стороны, если бы не вызывало реакцию его части с другой стороны от удара.
Если это действительно так то это очень странное поведение.
Поле электронное должно влиять само на себя думаю.
Аноним 27/10/21 Срд 05:20:28 548194453
Ещё вот что интересно, электрон неуничтожим, или это заряд неуничтожим, так-то заряд это электромагнитное поле, может электрон уничтожается, а затем если должно уничтожиться то что имеет заряд то заряд порождает электрон и улетает с ним от разрушившейся частицы.


Можно как-то увязать понятие заряд и электромагнитное поле?
Типа фотон он же электронейтрален, не плюс и не минус
Он переносчик взаимодействия, но заряд не переносит, переносят частицы..
С одной стороны фотон переносчик взаимодействия между заряженными частицами, с другой стороны сам заряд это электромагнитное поле, квант которого фотон.
Будто фотон переносчик взаимодействия между заряженными частицами, а электрон/позитрон переносчик взаимодействия между потенциалами электромагнитного поля
Единица переносчик двойке, а двойка переносчик единице.
Странно всё завязано. Будто и неправильно так рассматривать их.
Аноним 27/10/21 Срд 05:24:07 548195454
>>548193
>Как там описывается отсутствие деформации у электрона при столкновении? И других воздействиях.
Никак, ибо в квантовом случае понятия столкновения не существует. Это не шарики которые летают по траекториям. Даже понятие траектории не имеет смысла.
Ты вообще зря пытаешься разбираться над вещами из КЭД.
Во-первых тебе надо сначала разобраться с обычной квантовой механикой.
Во-вторых, все разговоры без формул это ничто. Язык не в состоянии передать адекватно уравнения движения из КЭД.
Аноним 27/10/21 Срд 05:33:21 548196455
>>548195
>Во-вторых, все разговоры без формул это ничто. Язык не в состоянии передать адекватно уравнения движения из КЭД.
Но ведь в результате эксперимента из установок выходят не формулки а совсем другое, картинки, или поток данных от плоско/объёмно расположенных условных датчиков.
Аноним 27/10/21 Срд 06:21:41 548199456
>>548192
>А он похоже неуничтожим
Вполне себе уничтожим. Деформируй его позитроном и капец будет.
Аноним 27/10/21 Срд 06:29:26 548201457
>>548194
Он в фотон превращается. Фотон улетает. Вот и всё.
Аноним 27/10/21 Срд 06:41:10 548202458
>>548194
>Типа фотон он же электронейтрален, не плюс и не минус
Ёбни фотоном об свинец со всей дури. Получиш электрон и позитрон. Электрон с минус зарядом и позитрон с плюс зарядом. Надо фотон помощнее взять. Рентгеновский подойдёт.

Ёбни электроном об позитрон. Получиш фотон. Отрицательный заряд электрона вычтется из положительного заряда позитрона. У фотона останется хуй вместо заряда.
Аноним 27/10/21 Срд 06:45:20 548204459
фотоны соснули
Аноним 27/10/21 Срд 08:41:38 548206460
>>548066
>>548033
>использовать подвижный ветряк/лопасти (всмысле, что бы он по ветру крутился) и передавать это на винт под водой.
Мельница вокруг своей оси крутится?
Аноним 27/10/21 Срд 13:35:33 548218461
>>548148
>1) Нахуя, где это свойство вылазит?
В экспериментах.
>2) Как у тебя волна может быть точечной, ты ебанулся?
Мы говорим о частицах, а не о волнах, долбоеб.
Аноним 27/10/21 Срд 14:53:38 548220462
>>548218
Электрон волна-частица.
Аноним 27/10/21 Срд 15:22:00 548221463
>>548220
Электрон - это элементарная частица.
Аноним 27/10/21 Срд 16:00:34 548222464
Почему собственное лицо постоянно выглядит по-разному? Ну или по-разному воспринимается?
Допустим, в один день лицо выглядит красивым, на следующий день оно выглядит уже уродливым. Или утром лицо выглядит некрасивым, но вечером того же дня уже выглядит красивым.
Как будто бы постоянно разные лица.
Почему так?
Аноним 27/10/21 Срд 16:51:03 548225465
Аноним 27/10/21 Срд 18:26:43 548226466
>>548222
утром - заплыло
вечером присушилось

вчера недоспал - мешки под глазами
сегодня выспался - мешков нет

поел сахара - ебло высыпало
на след день всё чисто

дохуя причин и настроение тоже сильно влияет наверно
Аноним 27/10/21 Срд 19:00:12 548227467
>>548225
Нет, это частица, а не волна.
Аноним 27/10/21 Срд 19:01:43 548228468
>>548227
Ведёт себя как волна, значит волна.
Аноним 27/10/21 Срд 19:12:59 548229469
>>548228
Нет, он ведет себя как элементарная частица, потому что он и есть элементарная частица.
Аноним 27/10/21 Срд 19:51:30 548230470
Аноним 27/10/21 Срд 20:21:33 548232471
>>548230
Лучше я с твоей мамашей поебусь.
Аноним 27/10/21 Срд 20:27:32 548235472
>>548232
Хорошо, замутите троечку с моей мамашей, она уже давненько мёртвая, клин клином вышибают. Может, адекватными станете.
Аноним 27/10/21 Срд 21:07:33 548236473
>>548235
Я ее воскрешу и выебу, а потом снова убью. Затем я приду за тобой. Ебать не буду, просто заебашу.
Аноним 27/10/21 Срд 21:26:59 548237474
image.png 343Кб, 650x366
650x366
>>548236
>Я ее воскрешу
Казалось бы, что может пойти не так.
Аноним 28/10/21 Чтв 11:09:09 548242475
А есть какие то подвижки, где таки находится сознание? Чтобы разночтений не было - ЧТО из мозга надо перенести в другое тело, чтобы вот моё "я" смотрело уже из тех глаз? Или это чисто софтовая вещь, не завязанная на конкретные структуры в голове?
Аноним 28/10/21 Чтв 11:12:05 548243476
>>547140

В основном считается, что точка. А то, что обычно считается размером ЧД, это область, в которой скорость убегания от этой точки >=c... Проверить было бы интересно, но никто не знает как.
Аноним 28/10/21 Чтв 11:12:28 548244477
>>548242
Возьме перцептрон. Что из него нужно перенесте в другой перцептрон, что бы он так же хуево распозновал котиков?
Все веса и все ребра.
Аноним 28/10/21 Чтв 11:17:39 548245478
>>547148

Проверяли в 1970ых или даже раньше, скорость фотонов от летящих с большой скоростью частиц не складывается, всё равно получается c. Видимо, какая-то фундаментальная проводимость пространства.

Мне другое интересно, про околосветовую ракету с фарами. От ракеты свет будет удаляться со скоростью c, а я со стороны буду видеть, что свет еле ползёт относительно ракеты. Спецы по СТО, там что-то типа мультиверса получается, или как это работает?
Аноним 28/10/21 Чтв 11:22:19 548246479
>>547158

Синусоиды как на картинке там нет. Сама ЭМ волна похуй свет или радиоволны, это что-то типа пузыря, который во все стороны расходится от источника. Синусоида там - потенциал на краю этого пузыря, если так можно выразиться. На радиоволнах это просто нагляднее намного.
Аноним 28/10/21 Чтв 11:27:11 548247480
>>547337

Органика + воздух при горении дают углекислый газ и воду. Газ улетает, вода испаряется и тоже улетает. То, что осталось, ты и наблюдаешь, потому что горит не идеально ровно.
Аноним 28/10/21 Чтв 11:30:42 548248481
>>548245
Обычный эффект Допплера же, разве нет?
Аноним 28/10/21 Чтв 11:32:22 548249482
>>548246
Почему нет? Там же [количество фотонов] таких синусоид, фотон же колеблется.
Аноним 28/10/21 Чтв 11:33:14 548250483
>>547499

А позитроны ты откуда возьмёшь? Если нагенеришь реактором, энергии всё равно получится меньше затраченной. А в окружающем мире их почти нет.
Аноним 28/10/21 Чтв 11:33:39 548251484
>>547347
Не важно, что ты увидишь или радар успеет зафиксировать.
Скорость движения может быть больше 2-й космической в атмосфере. У человечества нет средств для сбития таких тех в принципе.
Аноним 28/10/21 Чтв 11:37:24 548253485
>>548249

Туда-сюда в пространстве со своей частотой он не перемещается кмк, как на картинках рисуют.
Аноним 28/10/21 Чтв 12:05:32 548255486
>>548244

Распознавание это про вычисления. Перенесёшь всю нейросеть - просто скопируешь функционал. Наблюдатель что из себя представляет? Откуда, что вот я именно в этом теле?
Аноним 28/10/21 Чтв 13:42:50 548256487
Аноним 28/10/21 Чтв 14:40:16 548259488
>>548245
>Спецы по СТО, там что-то типа мультиверса получается, или как это работает?
Нет, всё нормально. Сам подумай, я смотрю со стороны, свет медленно ползет относительно тебя, я смотрю на тебя, а ты замедлен пиздец, у тебя время на корабле замедленно в дохуя раз, понятное дело, что для тебя этот ползущий свет летит вперед со скоростью С.
Аноним 28/10/21 Чтв 14:44:03 548260489
image.png 338Кб, 1280x720
1280x720
>>548242
>А есть какие то подвижки, где таки находится сознание?
В обществе. Это социальный фантомный конструкт, эмерджентно возникающий в животном даже не обязательно в примате с целью эгоистического продвижения своей собственной автономности.

Если общество признает своей интегральной частью бутылку мочи и выдаёт ей паспорт, то бутылка мочи обладает сознанием. Верно и обратное: если твоё правительство объявляет тебя нон-грата и дегуманизирует тебя, тебя убьют те, кто раньше тебе улыбался, и будут они считать себя хорошими, сознательными гражданами.
Аноним 28/10/21 Чтв 18:27:41 548264490
>>548246
>Сама ЭМ волна похуй свет или радиоволны, это что-то типа пузыря, который во все стороны расходится от источника.
>лазер делает отверстие в материале с размером почти в длину волны, по подсчётам и замерам 100% количества фотонов попадает в область проделывания отверстия
Да, конечно, во все стороны.... пузырь...
Аноним 28/10/21 Чтв 18:32:09 548266491
>>548245
>Спецы по СТО, там что-то типа мультиверса получается, или как это работает?
Да какое блять сто, электромагнитное поле это поле, волна это волна, какая разница с какой скоростью ты летишь над водой и бросаешь в неё камешки, делая волны, они всё равно будут расходиться с одиноковой скоростью, хоть ты на месте стоя в воду бросай камни, хоть летя над водой на самолёте.
Аноним 28/10/21 Чтв 19:03:07 548276492
>>548255
>Наблюдатель что из себя представляет? Откуда, что вот я именно в этом теле?
Этот вопрос не имеет ответа. Предположительно, у животных, особенно у социальных, появилаясь необходимость предугадывать поведение сородичей, чтобы более выгодно с ними взаимодействовать. Т.е. в мозге возник механизм, позволяющий худо-бедо "эмулировать" и анализировать чужое сознание. А теперь представь, что в какой-то момент этот механизм начал работать с собственным сознанием, с самим собой в том числе.
Аноним 29/10/21 Птн 09:21:57 548311493
XQhnLgiz.jpg 54Кб, 400x400
400x400
>>548242
>А есть какие то подвижки, где таки находится сознание?
https://www.jneurosci.org/content/41/8/1769
Контур в таламусе.

> Чтобы разночтений не было - ЧТО из мозга надо перенести в другое тело, чтобы вот моё "я" смотрело уже из тех глаз?
Что есть "твоё "я""? Всякая там память и (без)условные рефлексы, например, туда входят? Если да, то хуй знает, вон, у улитки РНК трансплантировали в другую, и как бы опыт исходной перенёсся.
https://www.eneuro.org/content/5/3/ENEURO.0038-18.2018

> Или это чисто софтовая вещь, не завязанная на конкретные структуры в голове?
Пенроуз его ведает. Может мы все вообще рудиментарные вырожденные остатки геонов (чёрных дыр из сгустков гравитационной или электромагнитной энергии), читай максимоны, которые скогерировали со всей прочей материей во Вселенной, пока та была сжата в первые мгновения Большого Взрыва. В конце концов, бактерии, похоже, квантово запутываются со светом, и ничего, не жалуются:
https://iopscience.iop.org/article/10.1088/2399-6528/aae224/meta

Да и перелётные птички квантмех юзают:
https://www.nature.com/articles/s41586-021-03618-9


А из тела (я продолжаю сослагательные наклонения) мы не вылетаем лишь потому, что почему-то существует некая более сильная запутанность с таким негэнтропийным инструментом, как миллиарды лет квазисохраняющаяся молекула ДНК (закон равенства по модулю энтропии замкнутой системы и интеграла негэнтропий её наблюдателей в мультиверсе, лол). Ну а в чужие тела не входим лишь потому, что нихуя об этом не помним (память-то у другого тела).
Короче, маняфантазировать пока-что можно во все поля, ничего вменяемого (экспериментально-проверяемого) у учёных пока мною не замечено.

Ну и в нагрузку, вот какое-то уныние про квалиа на хабре опубликовано, вдруг тебе оно надо:
https://habr.com/ru/post/570824/
Аноним 29/10/21 Птн 21:49:11 548332494
16354995250460.gif 5279Кб, 682x900
682x900
windmill-histor[...].jpg 133Кб, 352x558
352x558
image004.jpg 5Кб, 193x135
193x135
Shesternia-koni[...].jpeg 353Кб, 3872x2592
3872x2592
>>548206
>Мельница вокруг своей оси крутится?
Ну, башенные мельницы предполагали поворот крыши, помимо стандартного управления парусами.

Я до сих пор не вдупляю, что у тебя за проблема:
> Я представляю, как шестеренками или коленвалом передать энергию с неподвижного винта передать энергию на винт под водой. Но не могу представить, как со свободно вращающегося винта передать

Ну ок. Давай забьём на всё придуманное и сделаем по уебански просто.
Вот, на пике 3 передача винт-гайка.
Допустим, лопасти ветряка передают движение на коническую пару, один из конусов которой имеет внутри резьбу под этот самый винт, проходящий донизу твоего гидротехнического сооружения, а там через вторую такую же коническую пару вращает твою подводную лопасть (или что там у тебя). Верхняя часть башни поворачивается (вместе с вертикальной шестернёй верхней конической пары) и фиксируется ардуинкой и магнитным замком. В какой части данной схемы несоответствие твоим высоким стандартам транзита энергии движения?
Аноним 30/10/21 Суб 01:17:21 548340495
>>548242
Подвижки есть. Порядка сотен тысяч подвижек в год. Такое их количество само по себе препятствует пониманию.
Это Вавилонская башня 21 века.
Аноним 30/10/21 Суб 02:02:06 548341496
>>548311
>Да и перелётные птички квантмех юзают
Так можно дотрепаться, что все её юзают. По крайней мере, все у кого есть глаза. Потому что работу никакого источника света без квантовой физики объяснить нельзя.
Аноним 30/10/21 Суб 13:09:19 548359497
Если ожидаемая продолжительность жизни 80 лет, то почему у меня только из соседей умерли 2 человека до 40?
Аноним 30/10/21 Суб 14:25:52 548362498
16344592810870.jpg 1288Кб, 1247x898
1247x898
>>548359
>Если ожидаемая продолжительность жизни 80 лет, то почему у меня только из соседей умерли 2 человека до 40?
В экзистенциальном Аду (откуда ты капчуешь) год за два.
Очевидно же.
Аноним 30/10/21 Суб 14:45:03 548363499
>>548359
Она не ожидаемая, она средняя. Так что надо смотреть не только на тех соседей кто помер, но и на тех кто выжил.
Аноним 30/10/21 Суб 16:00:31 548364500
>>548363
>не только на тех соседей кто помер, но и на тех кто выжил.
Конечно, ведь тех кто живёт до 90-120 лет в рашке очень много.
Аноним 30/10/21 Суб 16:25:14 548365501
>>548364
В рашке ожидаемая у хуемразей 68, а не 80. А таких вполне себе много, просто ты с ними скорее всего и не пересекаешься.
Аноним 30/10/21 Суб 16:57:24 548366502
15926672849280.png 329Кб, 500x681
500x681
Не нашел раздела "механика", попробую спросить тут.

Можете объяснить чайнику, чем плоха цепная передача и почему ее на машинах заменили на вал? Слышал, что так меньше потери. А почему?
Аноним 30/10/21 Суб 17:03:08 548367503
Объясните, зачем в вузах существуют философские факультеты?
Какая от них практическая польза?
Я бы разогнал всех этих бесполезных философов, лучше готовить больше технарей.
Аноним 30/10/21 Суб 17:16:24 548368504
>>548366
Чем длиннее цель, тем больше у нее потерь.
Цепи дороже по производству и эксплуатации.
И у цепи больше структурных элементов, а это значит больше точек для отказа.
Но главная причина использования вала в том, что для него не критично перекос оси. Для цепи шатание оси очень критично.
Аноним 30/10/21 Суб 17:57:03 548369505
16324532045690.jpg 11Кб, 392x370
392x370
>>548367
>Объясните, зачем в вузах существуют философские факультеты?
Оттуда сравнительно массово забирают в чиновники.
Моё маняИМХО, почему: власти считают таких выпускников в меру безопасными, чтобы те замутили какую-нибудь реально дупоподрывающую технологию аля блокчейн, и в меру полезными, чтобы делать изобретательные отписки или публично объяснять, почему обосралось не конкретно их ведомство, а какое-то из соседних.

Как считаешь, выгодно ли ВУЗу иметь чиновников-выпускников? Или им интересней узнавать что-то новое о себе из повесток в мировой суд?

> Какая от них практическая польза?
Оповещать (на аутсорсе) специализированные службы о возникновении новых или изменении старых идеологических веяний, распространении уже имеющихся в таких-то социальных выборках.

Допустим, стали барнаульцы массово гей-оргии устраивать.
И вот младой бакалавр, в целях защиты диссертации, внедряется туда и пишет сводку, какие отголоски древнегреческой культуры Спарты возникают на современном отечественном пространстве. Его научрук стучит, к бакалавру прилетает повесточка с подозрением на ЛГБТ-пропаганду несовершеннолетним, в гей-ресторане/отделении выясняют все явки/пароли, присовокупляют к уже имеющемуся компромату какой-нибудь несанкционированный гомопикет в поддержку Навального у входа в здание, строят козни дальше (или накрывают группу с публичным заявлением о предотвращении готовящегося скрепоподрыва агентами ГосДепа). Ну, это в идеальном случае, конечно. Чаще просто ретроспективно сравнивается политическая идеология, чтобы в случае резкой смены политического курса пропагандонам было легче ориентироваться, на какие позитивные аспекты старого нового курса надо акцентировать внимание, а какие стоит опустить, чтобы не получить сравнение с Гитлером. Ну, там, Океания всегда поддерживала Талибан и всё такое...

> Я бы разогнал всех этих бесполезных философов, лучше готовить больше технарей.
Ну вот в Совке наготовили кучу инженеров. Это им не особо помогло торговать на вокзалах жвачкой в 90-ые. У тебя может быть 1.0E+8 технарей, но если над ними будут стоять не понимающие мировой и отечественной политической конъюнктуры управленцы (а технари так раз чаще всего в этом плавают как говно в проруби), то будут эти все технари на одну восьмую ставки работать на китайские заводы по производству фаллоимитаторов, делая чертежи по поиску точки Джи.
Короче, философы-гуманитарии нужны так раз для того, чтобы заводы с технарями в них не продавались по совсем уж бросовым ценам заморским баринам.
Аноним 30/10/21 Суб 17:58:31 548370506
>>548367
>Какая от них практическая польза?
Руководить технариками.
Аноним 30/10/21 Суб 18:33:11 548372507
>>548370
Не видел ни одного начальника с дипломом философа.
Они или идут в макдональдс, или остаются профессорами на том же факультете - т.е. являются бесполезными паразитами, существующими на наши налоги. И зачем мне их кормить?

>Оттуда сравнительно массово забирают в чиновники.
Примеры? У чиновников свои факультеты. У депутатов обычно юридическое образование.
Философ это хуйня без задач, если в совке из них готовили идеологов марксизма, то сейчас они не используются государством. А роль отдела пропаганды отошла к РПЦ.

>Оповещать (на аутсорсе) специализированные службы о возникновении новых или изменении старых идеологических веяний, распространении уже имеющихся в таких-то социальных выборках.
Это задача социологов и политологов, а не философов.

>Ну вот в Совке наготовили кучу инженеров. Это им не особо помогло торговать на вокзалах жвачкой в 90-ые.
Если только тупые, которые поступали по блату. Все нормальные инженеры ушли в IT.

>Короче, философы-гуманитарии нужны так раз для того, чтобы заводы с технарями в них не продавались по совсем уж бросовым ценам заморским баринам.
СССР был развален сверху, и ни один философ-марксист не почесался. Философы не могут быть оппозицией, они всегда подстилки текущего режима. Если сейчас заводы пилятся на металл, то дегенерат Дугин говорит, что это правильно. Если это СССР, то там философы топили за руководящую роль партии. Если это западные универы, то там они продвигают повесточку про тысячи гендеров. Но эти же самые функции может делать лахта за 15 рублей в день, безо всякого образования кроме 9 классов, набираемая с улицы.
Так зачем нужны философы обществу? Вот есть, к примеру, уфологи, в вузах же нет факультетов уфологии. От философии польза такая же, как от уфологии.
Аноним 30/10/21 Суб 19:17:30 548373508
>>548367
> бесполезных философов,

Зачем ты так о всех философах? Вот дяденька-философ:

https://en.wikipedia.org/wiki/Steve_Awodey

написал весьма годную книгу по теории категорий.
Не всё философы бесполезны.
Аноним 30/10/21 Суб 19:36:04 548374509
>>548372
>Философы не могут быть оппозицией, они всегда подстилки текущего режима. Если сейчас заводы пилятся на металл, то дегенерат Дугин говорит, что это правильно. Если это СССР, то там философы топили за руководящую роль партии.
:s/философы/интеллигенция/g
Потому что тебя иначе ссылали или расстреливали, зая сладкая.
Аноним 30/10/21 Суб 20:06:07 548375510
>>548369
По огромной куче высранного говна сразу узнаётся филосов.
Аноним 30/10/21 Суб 20:09:29 548376511
Аноним 30/10/21 Суб 20:09:59 548377512
>>548276

Ну, хотя бы так. Спс :)
Аноним 30/10/21 Суб 20:10:56 548378513
>>548266

Отличная аналогия, дошло.
Аноним 30/10/21 Суб 20:22:48 548379514
>>548359

У меня ттт нет знакомых умерших так рано. За 70 минимум. Много за 80.
Аноним 31/10/21 Вск 06:45:17 548390515
Бля, эти пидоры процессорные совсем охуели, фокуссируют луч лазера в толщину практически одной длины волны.
Аноним 31/10/21 Вск 11:19:37 548395516
В чем состоит сложность в создании всё более мощных квантовых компьютеров? С обычными понятно, там уже все упирается в масшатабы, а с этими что? Они там атомы вручную что ли расставляют?
Аноним 31/10/21 Вск 17:15:57 548404517
Растворённый в воде сахар остаётся твёрдым веществом? Что вообще происходит с веществом при растворении? Куда девается межфазная граница?
Аноним 31/10/21 Вск 19:09:31 548408518
>>548404
Решетка сахара распадается, молекулы воды растаскивают молекулы сахара и окружают их.
Аноним 31/10/21 Вск 20:29:38 548410519
>>548408
Вау, я наконец-то представил это в голове, это круто. А как так же наглядно понять вообще всю классическую механику, с углублением в сплошные среды и твёрдые тела? Что прочитать, чтобы представить модель в общем, со стороны эпирики, чтобы потом уже в погружаться в матан.
Аноним 31/10/21 Вск 22:17:43 548411520
>>548368
Спасибо. Т.е. именно большей "потери" на цепи нет? Там проблема в надежности и сложности?
Аноним 31/10/21 Вск 23:37:03 548417521
1635712616518.png 2331Кб, 2340x1080
2340x1080
>>547120 (OP)
Математики на месте?

Для одного из достижений требуется собрать все 40 растений.

В магазине можно купить четыре типа ростков:
Обычный. 7500$. Рандомное растение.
Бронзовый. 15000$. 25% шанса на отсутствующее.
Серебрянный. 30000$. 50%.
Золотой. 50000$. 100%.

Что наиболее рационально покупать исходя из соотношения цена/шанс?
Аноним 01/11/21 Пнд 00:22:30 548419522
IMG202110311923[...].jpg 116Кб, 640x640
640x640
>>548417
До 31 бери по 7500, до 38 по 15000, прследние 2 бери по 50000
Мимопролетарий
Аноним 01/11/21 Пнд 03:56:55 548422523
>>548417
Вообще всё это выглядит как лоховской развод.
С бронзовыми и серебряными высока вероятность потратиться минимально. С бронзовыми чуть дешевле, но заебно. С серебрянными чуть дороже, зато быстрее. Но разница будет в пределах статистической погрешности.
С обычными шансы собрать околонулевые.
с золотыми получится быстрее всего, но переплатишь примерно на 10% больше, чем с серебряными или бронзовыми.
Это я так приблизительно почувствовал.
Аноним 01/11/21 Пнд 13:48:05 548438524
>>548417
Покупать стоит либо обычные либо золотые. Если рисковать не хочешь. Золото в среднем более выгодно, но на серебряных/бронзовых может повезти получить больше. А может и не повезти и проебешь.
Обычные семена покупаешь пока не наберешь 34 штуки, потом переходишь на золото.
Аноним 01/11/21 Пнд 14:10:48 548439525
image.png 63Кб, 506x698
506x698
>>548417
Вот составил таблицу вероятностей в экселе. 1-й столбец - количество уникальных цветков (сначала ноль). 2-й соответственно сколько таких осталось собрать. 3-й вероятность по убывающей заполучить уникальный цветок с первого раза при покупке обычного ростка. 4-й вероятность потратить больше 7500, 5-й больше 15000. Это всё при работе именно с обычным ростком.
А насчёт рациональности, это надо уже самому определять, что для тебя рациональность. По идее первой покупкой очевидно берём обычный - у него 100%. Далее у тебя есть выбор: с 97.5 % взять уникальный цветок за 7500 или со 100% за 50000, купив золотой росток. Тут надо вводить какое-то дополнительное значение, учитывающее и цену покупки, и вероятность успеха. А там уже в том же экселе можно красиво будет допилить.
Аноним 01/11/21 Пнд 15:30:25 548441526
H225972c495d64f[...].jpg 10Кб, 300x300
300x300
Как работает такая спиральная пружина? Изгиб, растяжение, торсион (кручение)?
Аноним 02/11/21 Втр 05:15:17 548463527
Настройки X
Ответить в тред X
15000
Макс объем: 40Mб, макс кол-во файлов: 4
Кликни/брось файл/ctrl-v
Стикеры X
Избранное / Топ тредов